Download as pdf or txt
Download as pdf or txt
You are on page 1of 294

Date: 12/05/2024

Class: Crash Course 12 hour Session

Syllabus Topic: Review of Syllabus

Title: Review on all CSEC Maths Sections

Sections of the CSEC Maths Syllabus

There are 9 sections of the CSEC Maths Syllabus.

SECTION 1 – Number Theory and Computation

SECTION 2 – Consumer Arithmetic

SECTION 3 – Sets

SECTION 4 – Measurement

SECTION 5 – Statistics

SECTION 6 – Algebra

SECTION 7 – Relations, Functions and Graphs

SECTION 8 – Geometry and Trigonometry

SECTION 9 – Vectors and Matrices

1
TABLE OF CONTENTS

Topic Page Number

Section 1 – Number Theory and Computation 3

Section 2 – Consumer Arithmetic 26

Section 3 - Sets 42

Past Paper Questions 48

Section 4 - Measurement 63

Past Paper Questions 84

Section 5 – Statistics 94

Past Paper Questions 125

Section 6 – Algebra 134

Past Paper Questions 164

Section 7 – Relations, Functions and Graphs 171

Past Paper Questions 204

Section 8 – Geometry and Trigonometry 221

Past Paper Questions 254

Section 9 – Vectors 260

Past Paper Questions 268

Section 9 - Matrices 276

Past Paper Questions 285

2
Section 1 - Number Theory and Computation

Objective 1: Distinguish among sets of numbers.

Firstly, you should know,


{} means the set of
… means and so on (continuing in the same pattern)
∈ means is a member of (belongs to the set of)

Name Symbol Notation for CSEC

Natural Numbers ℕ {1, 2, 3, … }

Whole Numbers 𝕎 {0, 1, 2, 3, … }

Integers ℤ {… , −3, −2, −1, 0, 1, 2, 3, … }

Rational ℚ 𝑝
where 𝑝, 𝑞 ∈ ℤ and 𝑞 ≠ 0
𝑞

Irrational 𝕀 or ℚ′

Real ℝ

Venn Diagram:



ℚ′

𝕎

Take note of the inclusion relations, for example, ℕ ⊂ 𝕎 ⊂ ℤ ⊂ ℚ ⊂ ℝ.

3
You should have an understanding of the sequences of numbers that have a recognisable

pattern.

Sequence of Numbers Note

Factors A factor is a number that divides the given number

without any remainder.

Eg. The factors of 12 are 1, 2, 3, 4, 6, 12.

Multiples A multiple is a product that we get when one number is

multiplied by another number.

Eg. The multiples of 5 are 5, 10, 15, 20, 25, …

Square Numbers A square number is the result when a number is

multiplied by itself.

Eg. 1, 4, 9, 16

Even Numbers Even numbers can be divided by two without leaving a

remainder.

Odd Numbers Odd numbers cannot be divided by two without leaving a

remainder.

Prime Numbers Prime numbers are numbers that can be divided by other

factors besides 1 and itself.

Composite Numbers A composite number is a positive integer that can be

formed by multiplying two smaller positive integers.

4
Objective 2: Compute powers of real numbers of the form 𝑥 𝑎 , where 𝑎 ∈ ℚ.

You are required to know the following laws of indices:

𝑥1 = 𝑥

𝑥0 = 1

𝑥 𝑚 ∙ 𝑥 𝑛 = 𝑥 𝑚+𝑛

𝑥𝑚
= 𝑥 𝑚−𝑛
𝑥𝑛

(𝑥 𝑚 )𝑛 = 𝑥 𝑚𝑛

(𝑥𝑦)𝑚 = 𝑥 𝑚 𝑦 𝑚

𝑥 𝑚 𝑥𝑚
(𝑦 ) = 𝑦 𝑚

1
𝑥 −𝑚 = 𝑥 𝑚

5
Objective 3: Evaluate numerical expressions using any of the four basic operations on real

numbers.

This include addition multiplication, subtraction and division of whole numbers,

fractions and decimals; order of operations.

BODMAS is an acronym used to help understand the order in which operations take place

when finding the solution to a question.

Brackets Of Division Multiplication Addition Subtraction

6
Objective 4: Convert among fractions, percents and decimals.

The table below shows the equivalence between fractions, decimals and percent.

Fraction Decimal Percent

1 0.5 50%
𝟏
2
Family
𝟐
2 1 100%
2

1 1
0.333 (to 3 d.p.) 33 3%
3

2 2
𝟏 0.667 (to 3 d.p.) 66 3%
Family
𝟑 3

3 1 100%
3
1
0.25 25%
4

2
0.5 50%
4
𝟏
Family
𝟒
3
0.75 75%
4

4
1 100%
4

1
0.2 20%
5

2
0.4 40%
5

𝟏 3
Family 0.6 60%
𝟓 5

4
0.8 80%
5

5 1 100%
5

7
1
0.125 12.5%
8

2
0.25 25%
8

3
0.375 37.5%
8

4
0.5 50%
8
𝟏
Family
𝟖
5 0.625 62.5%
8

6
0.75 75%
8

7
0.875 87.5%
8

8
1 100%
8

1
0.1 10%
10

2
0.2 20%
10

3
0.3 30%
10

4
0.4 40%
10

5 0.5 50%
10
𝟏
Family
𝟏𝟎
6
0.6 60%
10

7
0.7 70%
10

8
0.8 80%
10

9
0.9 90%
10

10
1 100%
10

8
Objective 5: List the set of factors and multiples of a given integer.

A factor is a number that divides another number leaving no remainder.

You should be able to state the positive and negative factors of an integers.

Suppose the integer is 12.

The positive factors are {1, 2, 3, 4, 6, 12}.

The negative factors are {−1, −2, −3, −4, −6, −12}.

A multiple is the product result of one number multiplied by another number.

Suppose the integer is 3.

The multiples of 3 are {3, 6, 9, 12, 15, 18, … }.

9
Objective 6: Compute the HCF or LCM of two or more positive integers.

The Highest Common Factor (HCF) is the highest number among all the common factors

of the given numbers.

Eg. The HCF of 12 and 18 is 6.

The Lowest Common Multiple (LCM) is the lowest number among all the common

multiples of the given numbers.

Eg. The LCM of 12 and 18 is 36.

10
Objective 7: State the value of a digit of a numeral in a given base.

Tips:

1. When placing numbers from the decimal system to another base, you perform

factorization until you reach zero.

2. Ensure to write your answer starting from the bottom to the top.

Example :

Converting 1210 to Base2 .

Solution:

2 12
2 6 r0 Note:

2 3 r0 Notice how we write the answer


from the last remainder to the first!
2 1 r1
0 r1

∴ 1210 to Base2 = 11002

Suppose we wanted to go backwards.

In order words, converting a number from base 2 to base 10.

11
Example 2:
Convert 110002 to Base10

Solution:

24 = 16 23 = 8 22 = 4 21 = 2 20 = 1

1 1 0 0 0

𝟏𝟔 𝟖 𝟎 𝟎 0

Sum of numbers in Row 3 = 16 + 8 + 0 + 0 + 0


= 24

∴ 110002 to Base10 = 2410

12
Objective 8: Convert from one set of units to another.

Conversion scales that you should know are:

1 𝑐𝑚 = 10 𝑚𝑚

1 𝑚 = 100 𝑐𝑚

1 𝑘𝑚 = 100 𝑚

1 𝑘𝑚 = 100 000 𝑐𝑚

Area scale = (linear scale)2

1 𝑚2 = 10 000 𝑐𝑚2

1 𝑘𝑚2 = (100 000)2 𝑐𝑚2

1 litre = 1000 𝑐𝑚3

You should be able to convert from 12-hour clock to 24-hour clock and vice-versa.

Some examples are given in the table below:

12 hour 24 hour

8:00 am 0800

1:00 pm 1300

4:30 pm 1630

8:00 pm 2000

13
Objective 9: Express a value to a given number of:

(a) significant figures

(b) decimal places

You can be asked to place a number to 1, 2 or 3 significant figures.

Examples:

0.1745 = 0.2 (to 1 significant figure)

0.1745 = 0.17 (to 2 significant figures)

0.1745 = 0.175 (to 3 significant figures)

1305.26 = 1000 (to 1 significant figure)

1305.26 = 1300 (to 2 significant figures)

1305.26 = 1310 (to 3 significant figures)

You can be asked to place a number to 0, 1, 2 or 3 decimal places.

Examples:

23.4652 = 23 (to the nearest whole number)

23.4652 = 23.5 (to 1 decimal place)

23.4652 = 23.47 (to 2 decimal places)

23.4652 = 23.465 (to 3 decimal places)

14
Question:
1
2−1
7
Consider 5 .
6

State the answer using the following:

(a) exact value

(b) to 3 decimal places

(c) to 3 significant figures

Solution:
1
2−1 36
7
(a) 5 = 35
6

1
2−1
7
(b) 5 = 1.029 (to 3 decimal places)
6

1
2−1
7
(c) 5 = 1.03 (to 3 significant figures)
6

15
Objective 10: Use properties of numbers and operations in computational tasks.

Additive identity

The additive identity is 0.

Adding 0 to a number does not change it.

𝑎+0= 0+𝑎 = 𝑎

Multiplicative Identity

The multiplicative identity is 1.

Multiplying a number by 1 does not change it.

𝑎×1= 1×𝑎 =𝑎

Additive Inverse

The additive inverse of a number 𝑎 is −𝑎.

𝑎 + (−𝑎) = 0

For example,

The additive inverse of 2 is −2.

Multiplicative Inverse
1
The multiplicative inverse of a number 𝑎 is 𝑎.

1
𝑎×𝑎 =1

For example,
1
The multiplicative inverse of −3 is − 3.

16
Commutativity

➢ Addition and multiplication follow the rule of commutativity.

➢ Subtraction and division do NOT follow the rule of commutativity.

➢ The commutative law asks the question: Is order important?

➢ 𝑎+𝑏 =𝑏+𝑎

➢ 𝑎×𝑏 =𝑏×𝑎

Associativity

➢ Addition and multiplication follow the rule of associativity.

➢ Subtraction and division do NOT follow the rule of associativity.

➢ This law deals with the grouping of the numbers and letters.

➢ (𝑎 + 𝑏) + 𝑐 = 𝑎 + (𝑏 + 𝑐)

➢ 𝑎 × (𝑏 × 𝑐) = (𝑎 × 𝑏) × 𝑐

Distributivity

➢ This law deals with the multiplying everything inside parentheses by what is

outside it.

➢ Multiplication distributes over addition.

➢ 𝑎(𝑏 + 𝑐) = 𝑎𝑏 + 𝑎𝑐

➢ Example: 3(4 + 5) = 3(4) + 3(5)

Example: 3(4 + 5) = 12 + 15

Example: 3(4 + 5) = 27

17
Objective 11: Write any rational number in scientific notation.

Scientific Notation is of the form : 𝑨 × 𝟏𝟎𝑩

• where 1 ≤ 𝑨 < 10

i.e. 𝐴 is always a number between 1 and 10, not including 10

• and where 𝑩 ∈ ℤ

i.e. 𝐵 is always an integer (either negative or positive)

Tip:
When the decimal point moves to the left, the value of B becomes positive.
When the decimal point moved to the right, the value of B becomes negative.

Example:

Place the following in scientific notation:

(a) 4 600 000

(b) 0.0023

Solutions:

(a) 4 600 000 = 4.6 × 106

(b) 0.0023 = 2.3 × 10−3

18
Objective 12: Calculate any fraction or percentage of a given quantity.

Objective 13: Express one quantity as a fraction or percentage of another.

Objective 14: Compare quantities.

There are 3 ways a question can come.

Initial Final Percentage (%) Method

1. • Get difference

𝑑𝑖𝑓𝑓𝑒𝑟𝑒𝑛𝑐𝑒
• Put × 100
𝐼𝑛𝑖𝑡𝑖𝑎𝑙

2. • % difference × initial

• add to initial

3. • 100% ± % difference

• Find for one

• Find for 100%

19
Objective 15: Order a set of real numbers.

You should be able to rearrange a set of real numbers in ascending order to descending
7 7
order. For example, 1.1, 2 , √2 , 1.45, 𝜋 in ascending order is 1.1, √2 , 1.45, 𝜋 , 2 .

20
Objective 16: Compute terms of a sequence given a rule.

Objective 17: Derive an appropriate rule given the terms of a sequence.

Consider the sequence 2, 4, 6, 8, 10, …. .

The terms in the sequence can be generated by using the rule which takes each term and

adds 2 to the previous term.

An expression for this rule will be 2𝑛.

Consider the sequence 5, 8, 11, 14, …. .

An expression for this rule will be 3𝑛 + 2.

Note: This can show up in Question 7 of the Paper 2.

Question:

Consider the following pattern:

Figure 1 Figure 2 Figure 3

(a) Draw Figure 4.

(b) Fill in the missing space in the table below:

21
Figure, 𝒏 Number of dots Number of sticks

1 1 4

2 2 7

3 3 10

⋮ ⋮ ⋮

Solution:

(a) Figure 4 is shown below:

(b) The completed table is as follows:

Figure, 𝒏 Number of dots Number of sticks

1 1 4

2 2 7

3 3 10

⋮ ⋮ ⋮

𝑛 𝑛 3𝑛 + 1

22
Objective 18: Divide a quantity in a given ratio.

Tips when doing questions in ratios:

→ Find the value of all parts.

→ Find for one part.

Question 1:

Suppose that Mr. Kim shared a total of $60 between his three children, John, Claire and

Mark in the ratio 2:3:5 respectively. How much did each child receive?

Solution:

Total parts = 2 + 3 + 5

Total parts = 10 parts

10 parts = $60

$60
1 part = 10

1 part = $6

Therefore,

John receives 2 × $6 = $12.

Claire receives 3 × $6 = $18.

Mark receives 5 × $6 = $30.

23
Question 2:

Kate and Reshma shared a bag of chocolates in the ratio 2 ∶ 5. Reshma receive 15

chocolates more than Kate. How many chocolates were in the bag initially?

Solution:

Total parts = 2 + 5

Total parts = 7 parts

Difference = 5 − 2

Difference = 3 parts

3 parts = 15 chocolates

15
1 part = 3

1 part = 5 chocolates

Hence,

Number of chocolates initially in the bag = 7 × 5

Number of chocolates initially in the bag = 35 chocolates

24
Objective 19: Solve problems involving concepts in number theory.

This includes ratios, rates and proportion.

Tips when doing questions in these topics:

→ Finding for one is a division process.

→ To find what you want, you multiply.

25
Section 2 – Consumer Arithmetic

Objective 1: Calculate

(a) discount

(b) sales tax

(c) profit

(d) loss

Discount - This is a reduction of the Marked Price.

Opposite of discount is called a mark-up.

Formula:

Discount = Marked Price – Selling Price

Sales Tax – This is an additional cost fixed by the government.

Profit Loss

This is a financial gain. This is when there is no financial gain.

There is a decrease of finances on a

transaction.

Profits occur when the Selling Price is Losses occur when the Selling Price is

more than the Cost Price. less than the Cost Price.

Formula: Formula:

Profit = Selling Price – Cost Price Loss = Cost Price – Selling Price

when Selling Price > Cost Price when Selling Price < Cost Price

26
Objective 2: Calculate

(a) percentage profit

(b) percentage loss

Percentage Profit

• The Percentage Profit is found by dividing the profit by Cost Price and

multiplying by 100.

• Formula:

𝑷𝒓𝒐𝒇𝒊𝒕
Percentage Profit = 𝑪𝒐𝒔𝒕 𝑷𝒓𝒊𝒄𝒆 × 𝟏𝟎𝟎

Percentage Loss

• The Percentage Loss is found by dividing the loss by Cost Price and multiplying

by 100.

• Formula:

𝑳𝒐𝒔𝒔
Percentage Loss = 𝑪𝒐𝒔𝒕 𝑷𝒓𝒊𝒄𝒆 × 𝟏𝟎𝟎

27
Objective 3: Express a profit, loss, discount, markup and purchase tax, as a percentage of

some value.

Tips:

1. Ensure to read your question carefully and perform the appropriate calculation.

When calculating discount, you subtract the amount of discount from the marked

price. When calculating markup or purchase tax, you add the amount of tax to the

marked price.

2. When finding the percentage of some value, ensure to place your amount over the

original value.

28
Objective 4: Solve problems involving marked price, selling price, cost price, profit, loss

or discount.

Recall the difference between marked price, selling price and cost price.

Marked Price/Displayed Price

• This is the amount that the store displays the item for.

Selling Price

• This is the amount that the customer paid the store for the item.

Cost Price

• This is the amount that the store pays for their item.

29
Objective 5: Solve problems involving payments by instalments as in the case of hire

purchase and mortgages.

• This is paying back a sum of money (or Lump sum) in equal parts over a period

of time.

• Hire Purchase, mortgages and loans all involve instalments

• Interest is the extra money paid in an instalment type of payment.

Formula:

Hire Purchase Price = Downpayments + Instalments

Formula:

Interest = Hire Purchase – Cash Price

𝑨𝒎𝒐𝒖𝒏𝒕 𝒐𝒇 𝑰𝒏𝒕𝒆𝒓𝒆𝒔𝒕
% Interest = × 𝟏𝟎𝟎%
𝑪𝒂𝒔𝒉 𝑷𝒓𝒊𝒄𝒆

Some persons might prefer a Hire Purchase plan instead of paying cash.

Reasons include:

• Does not have initial cash.

• Does not want to spend that money now.

• Can make use of the item right away and make a profit in his business.

• Build credit.

30
Objective 6: Solve problems involving simple interest.

Formula:

𝑷×𝑹×𝑻
Simple Interest = 𝟏𝟎𝟎

where

𝑃 = principal

𝑅 = rate (%)

𝑇 = time (years)

Depending on the variable you are trying to find and the information provided in the

question, the Simple Interest formula can be rearranged as follows.

𝑆𝐼 × 100
• Principal = 𝑅×𝑇

𝑆𝐼 × 100
• Rate = 𝑃×𝑇

𝑆𝐼 × 100
• Time = 𝑃×𝑅

To determine the total amount after receiving the interest, add the principal amount to

the simple interest.

Formula: Amount = Principal + Simple Interest

31
Question:

Timmy invested $36 000 into Edwards Bank. At the end of 10 years, he had $42 000.

(a) What is the simple interest on his investment?

(b) What is the rate of the interest at JEB (Josiah Edwards Bank)?

Solution:

(a) Simple Interest = Amount – Principal

Simple Interest = $42 000 − $36 000

Simple Interest = $6000

𝑆𝐼×100
(b) 𝑅 = 𝑃×𝑇

$6000×100
𝑅 = $36000 × 10

𝑅 = 1.67 % (to 3 significant figures)

32
Objective 7: Solve problems involving compound interest

Compound interest is when you earn interest on both the money you've saved and the

interest you earn.

Formula:

𝑹 𝒏
Amount = 𝑷 (𝟏 + )
𝟏𝟎𝟎

where

𝑃 = principal

𝑅 = rate as a percentage

𝑛 = number of years

There are two approaches when doing questions on Compound Interest:

1. Part by Part (year by year) – only used when given small time like 2 or 3 years.

2. Formula

33
Objective 8: Solve problems involving appreciation and depreciation

Definition of Appreciation:

Appreciation refers to when the value of something increases over time.

Examples include land (real estate), gold, antiques and fine art.

Formula:

𝑹 𝒏
Amount = 𝑷 (𝟏 + 𝟏𝟎𝟎)

where

𝑃 = principal

𝑅 = rate as a percentage

𝑛 = number of years

Definition of Depreciation:

Depreciation refers to when the value of something decreases over time.

Examples include cars and machinery.

Formula:

𝑹 𝒏
Amount = 𝑷 (𝟏 − )
𝟏𝟎𝟎

where

𝑃 = principal

𝑅 = rate as a percentage

𝑛 = number of years

34
Question:

James bought a car for $150 000. The value of the car depreciates by 10% each year.

How much is the car worth at the end of 2 years?

Solution:

Method 1: Year by year

After 1st year,

Value of car = 90% of $150 000

90 $150 000
Value of car = 100 × 1

Value of car = $135 000

After 2nd year,

Value of car = 90% of $135 000

90 $135 000
Value of car = 100 × 1

Value of car = $121 500

Method 2: Formula

𝑅 𝑛
𝐴 = 𝑃 (1 − 100)

10 2
𝐴 = 150 000 (1 − 100)

𝐴 = $121 500

∴ The value of the car at the end of 2 years is $121 500.

35
Objective 9: Solve problems involving measures and money

Tip:

1. Ensure to write appropriate statements in your solution.

2. When doing questions involving money, it may incorporate the idea of ratios. For

example, this can be seen in the topic of Exchange rate where you will use the

technique of “Finding for one”. Remember that finding for one is a division

process.

3. When doing questions on Exchange Rate, you should incorporate this three step

approach in your solution.

Step 1: Write down the rate. Put what you are trying to find on the left hand side.

6.75 TTD = 1 USD

Step 2: Find for one.


1
1 TTD = 6.75

Step 3: Multiply to find what you want.


1
1350 TTD = 6.75 × 1350

1350 TTD = 200 USD

36
Objective 10: Solve problems involving

(a) rates and taxes

(b) utilities

(c) invoices and shopping bills

(d) salaries and wages

(e) insurance and investments

Tips:

1. When calculating total electricity cost or total water cost, first find the variable

cost and then the fixed cost. Add both costs to find the total cost of the bill.

2. Look out for whether or not a late fee is applied to the total cost of the bill.

3. When finding new readings for electricity, always put the additional zeros in front

of your readings. For example, 00010310 kWh.

Question 1:

Consider the following information about an electricity bill.

Rental fee = $100

1 kWh = $0.065

Lewis used 1500 kWh for one month. What will be the amount on his electricity bill?

Solution:

Total fee = Fixed fee + Variable fee

Total fee = $100 + (1500 × $0.065)

Total fee = $197.50

37
Question 2:

Recall that for meter readings, it continues to increase.

Suppose that the meter reading for June 30th is 00250 kWh.

1000 kWh was used in February.

What will be the meter reading at the end of February?

Solution:

Meter reading at the end of February = 00250 + 1000

Meter reading at the end of February = 01250 kWh

Question 3:

Consider the shopping bill below:

Item Quantity Unit Price Total

vests 3 100 𝑨

pants 2 𝑩 $500

hoodie 𝑪 300 $900

Subtotal 𝑫

VAT(12.5%) 𝑬

Total 𝑭

Solution:

𝐴 = 3 × $100

𝐴 = $300

38
$500
𝐵= 2

𝐵 = $250

$900
𝐶= 300

𝐶=3

𝐷 = $300 + $500 + $900

𝐷 = $1700

𝐸 = 12.5% of $1700

12.5 $1700
𝐸= ×
100 1

𝐸 = $212.50

𝐹 = $1700 + $212.50

𝐹 = $1912.50

Question 4:

Ryan works for 40 hours at a rate of $10 per hour. Overtime is calculated at time and a

half. Ryan works for 42 hours in a particular week.

(a) Find his basic wage.

(b) How many hours did he work overtime?

(c) What is the overtime rate?

(d) Determine his overtime wage.

(e) Calculate the total amount Ryan worked for in this particular week.

39
Solution:

(a) Basic wage = Basic rate × Number of basic hours

Basic wage = $10 × 40

Basic wage = $400

(b) Number of overtime hours = 42 − 40

Number of overtime hours = 2 hours

(c) Overtime rate = 1.5 × Basic rate

Overtime rate = 1.5 × $10

Overtime rate = $15

(d) Overtime wage = Overtime rate × Number of overtime hours

Overtime wage = $15 × 2

Overtime wage = $30

(e) Amount = Basic wage + Overtime wage

Amount = $400 + $30

Amount = $430

Question 5:

Consider,

Gross Income = $100 000

Tax allowance = $20 000

Tax rate = 25%

40
Calculate:

(a) taxable income

(b) amount of tax

(c) net income

Solution:

(a) Taxable income = Gross Income − Tax allowance

Taxable income = $100 000 − $20 000

Taxable income = $80 000

(b) Amount of taxes = 25% of $80 000

25 $80 000
Amount of taxes = 100 × 1

Amount of taxes = $20 000

(c) Net Income = Gross − Taxes

Net Income = $100 000 − $20 000

Net Income = $80 000

41
Section 3 - Sets

Objective 1: Explain concepts relating to sets.

Definition:

Sets are denoted by curly braces {} and the objects or elements inside the set are

separated by commas.

For example, the set of all even numbers can be denoted as {2, 4, 6, 8, ...}.

Description of sets using words:

Consider the solution set to be {𝑥: 1 < 𝑥 ≤ 5}.

This set can be described as “the set of all numbers that are greater than 1 but less than

or equal to 5.

Membership of a set:

Consider the set 𝐴 = {1, 2, 3}.

The elements 1, 2 and 3 are members of the set.

This is usually denoted by ∈. Since 2 belongs to set 𝐴, we can write 2 ∈ 𝐴.

Cardinality of a set:

The cardinality of a set is the number of elements in the set.

It is denoted by the symbol. | |.

For example, the cardinality of the set 𝐴 = {1, 2, 3} is 3.

Finite and infinite sets:

A finite set is a set that has a specific number of elements.

42
An infinite set is a set that has an uncountable number of elements. For example, the set

of real numbers is an infinite set.

Universal set:

The universal set is the set that contains all possible elements.

It is denoted by the symbol 𝑈.

Empty set:

An empty set is a set that contains no elements.

It is denoted by the symbol ∅.

Complement of a set:

The complement set is the set of all elements that are outside of the set.

For example, consider 𝑈 = {1, 2, 3, 4, 5} and 𝐴 = {1, 2, 3}.

The complement of 𝐴, denoted by 𝐴′ = {4, 5}.

Subsets:

A set 𝐵 is a subset of set 𝐴 if all elements of 𝐵 are also elements of 𝐴.

Suppose 𝐴 = {1, 2, 3} and 𝐵 = {1, 2}, then we can say that 𝐴 is a subset of 𝐵, denoted by

𝐴 ⊂ 𝐵.

43
Objective 2: Represent a set in various forms.

There are three representations of a set you should be familiar with:

1. Description

The set 𝐴 comprising the first three natural numbers.

2. Set builder notation

𝐴 = {𝑥: 0 < 𝑥 < 4, 𝑥 ∈ ℕ}

3. Listing

𝐴 = {1, 2, 3}

44
Objective 3: List subsets of a given set.

You should be able to identify the subsets of a given set.

Suppose the set 𝑆 = {𝑎, 𝑏, 𝑐}.

The subsets are:

{𝑎} {𝑏} {𝑐}

{𝑎, 𝑏} {𝑏, 𝑐} {𝑎, 𝑐}

{𝑎, 𝑏, 𝑐} {}

You should be able to determine the number of subsets of a set with 𝒏 elements.

Suppose the set 𝑆 = {𝑎, 𝑏, 𝑐}.

Number of subsets = 2𝑛

Number of subsets = 23

Number of subsets = 8

45
Objective 4: Determine elements in intersections, unions and complements of sets.

You should be able to apply the following result:

𝒏(𝑨 ∪ 𝑩) = 𝒏(𝑨) + 𝒏(𝑩) − 𝒏(𝑨 ∩ 𝑩)

46
Objective 5: Describe relationships among sets using set notation and symbols.

Objective 6: Draw Venn diagrams to represent relationships among sets.

Objective 7: Use Venn diagrams to represent the relationships among sets.

Objective 8: Solve problems in Number Theory, Algebra and Geometry using concepts in

Set Theory.

47
Past Paper Question – January 2009 – Paper 2 – Question 1

(a) Calculate the EXACT value of


3
3
4
1 5
2 −
3 6

giving your answer as a fraction. [3]

(b) BDS$ means Barbados dollars and EC$ means Eastern Caribbean dollars.

(i) Karen exchanged BDS$2000.00 and received EC$2700.00. Calculate the

value of one BDS$ in EC$. [2]

(ii) If Karen exchanged EC$432.00 for BDS$, calculate the amount of BDS$ which

Karen would receive. [2]

[Assume that the buying and selling rates are the same.]

(c) A credit union pays 8% per annum compound interest on all fixed deposits. A

customer deposited $24 000 in an account. Calculate the TOTAL amount of money

in the account at the end of two years. [4]

Total: 11 marks

48
Past Paper Question – January 2009 – Paper 2 – Question 1 – Solution

(a) Calculate the EXACT value of


3
3
4
1 5
2 −
3 6

giving your answer as a fraction. [3]

3
3 5
4
Using a calculator, 1 5 =2.
2 −
3 6

(b) BDS$ means Barbados dollars and EC$ means Eastern Caribbean dollars.

(i) Karen exchanged BDS$2000.00 and received EC$2700.00. Calculate the

value of one BDS$ in EC$. [2]

BDS$2000.00 = EC$2700.00
2700
BDS$1.00 = 2000

BDS$1.00 = EC$1.35

(ii) If Karen exchanged EC$432.00 for BDS$, calculate the amount of BDS$ which

Karen would receive. [2]

[Assume that the buying and selling rates are the same.]

BDS$1.00 = EC$1.35

EC$1.35 = BDS$1.00
1
EC$1.00 = 1.35

1
EC$432.00 = 1.35 × 432

EC$432.00 = BDS$320

49
(c) A credit union pays 8% per annum compound interest on all fixed deposits. A

customer deposited $24 000 in an account. Calculate the TOTAL amount of money

in the account at the end of two years. [4]

Using the compound interest formula,

𝑅 𝑛
𝐴 = 𝑃 (1 + 100)

8 2
𝐴 = 24000 (1 + 100)

𝐴 = $27 993.60

∴ The total amount of money in the account at the end of two years is $27 993.60.

Total: 11 marks

50
Question Bank - Sets

1. If 𝐴 = {𝑎, 𝑏, 𝑐}, then the number of subsets of 𝐴 is

(A) 2

(B) 3

(C) 4

(D)8

2. If 𝑃 = { }, then the number of subsets of 𝑃 is

(A) 0

(B) 1

(C) 2

(D)3

3. If 𝑃 = {𝑥}, then the number of subsets of 𝑃 is

(A) 0

(B) 1

(C) 2

(D)3

4. If the universal set 𝑈 = {𝑝, 𝑞, 𝑟, 𝑠, 𝑡, 𝑢} and 𝐻 = {𝑞, 𝑟, 𝑠}, then 𝐻 ′ =

(A) {𝑝, 𝑞, 𝑟}

(B) {𝑞, 𝑟, 𝑠}

(C) {𝑝, 𝑡, 𝑢}

(D){𝑝, 𝑞, 𝑟, 𝑠, 𝑡, 𝑢}

51
5. Which of the following Venn diagrams below is the region 𝑃′ ∩ 𝑄 shaded?

6. What is the region shaded in the diagram below?

(A) 𝐴 ∩ 𝐵

(B) 𝐴 ∪ 𝐵′

(C) 𝐵′

(D)𝐴′

52
7. If the universal set, 𝐴 = {𝑤ℎ𝑜𝑙𝑒 𝑛𝑢𝑚𝑏𝑒𝑟𝑠 𝑓𝑟𝑜𝑚 1 𝑡𝑜 10 𝑖𝑛𝑐𝑙𝑢𝑠𝑖𝑣𝑒},

𝐸 = {𝑒𝑣𝑒𝑛 𝑛𝑢𝑚𝑏𝑒𝑟𝑠}

and 𝑃 = {𝑝𝑟𝑖𝑚𝑒 𝑛𝑢𝑚𝑏𝑒𝑟𝑠},

then 𝑛(𝐸 ∩ 𝑃) =

(A) 0

(B) 1

(C) 2

(D)3

8. 𝑃 = {𝑝𝑟𝑖𝑚𝑒 𝑛𝑢𝑚𝑏𝑒𝑟𝑠}

𝐸 = {𝑒𝑣𝑒𝑛 𝑛𝑢𝑚𝑏𝑒𝑟𝑠}

𝑂 = {𝑜𝑑𝑑 𝑛𝑢𝑚𝑏𝑒𝑟𝑠}

Which of the following is an empty set?

(A) 𝑃 ∩ 𝐸 = ∅

(B) 𝑃 ∩ 𝑂 = ∅

(C) 𝑃′ ∩ 𝐸 = ∅

(D)𝑂 ∩ 𝐸 = ∅

9. If 𝑃 and 𝑄 are two sets where 𝑛(𝑃 ∩ 𝑄) = 16 and 𝑛(𝑃 ∩ 𝑄) = 6 and 𝑛(𝑃) = 9,

then 𝑛(𝑄 ) =

(A) 1

(B) 7

(C) 13

(D)25

53
Questions 10 and 11 refer to the diagram below.

The Venn Diagram below indicates the number of students in each set.

𝑈 = {𝑠𝑡𝑢𝑑𝑒𝑛𝑡𝑠 𝑖𝑛 𝑐𝑙𝑎𝑠𝑠}

𝑃 = {𝑠𝑡𝑢𝑑𝑒𝑛𝑡𝑠 𝑠𝑡𝑢𝑑𝑦𝑖𝑛𝑔 𝑃ℎ𝑦𝑠𝑖𝑐𝑠}

𝐺 = {𝑠𝑡𝑢𝑑𝑒𝑛𝑡𝑠 𝑠𝑡𝑢𝑑𝑦𝑖𝑛𝑔 𝐺𝑒𝑜𝑔𝑟𝑎𝑝ℎ𝑦}

10. How many students do NOT study Geography?

(A) 4

(B) 11

(C) 15

(D)18

11. How many students are there in the class?

(A) 18

(B) 23

(C) 24

(D)25

54
12. If 𝑄 = {𝑎, 𝑏, 𝑐, 𝑑, 𝑒}, how many subsets can be obtained from the set 𝑄?

(A) 1

(B) 5

(C) 52

(D)32

Questions 13 to 15 refer to the diagram below:

In the INCOMPLETE Venn Diagram below,

𝑈 = {𝑛𝑢𝑚𝑏𝑒𝑟𝑠 𝑓𝑟𝑜𝑚 𝑜𝑛𝑒 𝑡𝑜 𝑡𝑒𝑛 𝑖𝑛𝑐𝑙𝑢𝑠𝑖𝑣𝑒}

𝑃 = {𝑝𝑟𝑖𝑚𝑒 𝑛𝑢𝑚𝑏𝑒𝑟𝑠}

𝐸 = {𝑒𝑣𝑒𝑛 𝑛𝑢𝑚𝑏𝑒𝑟𝑠}

13. What number goes in 𝑃 ∩ 𝐸?

(A) 0

(B) 1

(C) 2

(D)3

55
14. What is the value of 𝑛(𝑃′ )?

(A) 4

(B) 5

(C) 6

(D)7

15. What is the value of 𝑛(𝑃 ∪ 𝐸)?

(A) 2

(B) 3

(C) 7

(D)8

16. Which of the following sets is defined by {𝑥 ∈ ℤ: − 3 < 𝑥 ≤ 3}?

(A) {1, 2, 3}

(B) {0, 1, 2, 3}

(C) {−2, −1, 0, 1, 2, 3}

(D){−3, −2, −1, 0, 1, 2, 3}

17. In a class of 23 students, 11 study Physics and 17 study Geography. What is the

least number of students that both study Physics and Geography?

(A) 3

(B) 4

(C) 5

(D)6

56
Answer 18 and 19 using: 𝑀 = {𝑎, 𝑏, 𝑐} , 𝑁 = {𝑏, 𝑐} and 𝑃 = {𝑎, 𝑒, 𝑖, 𝑜, 𝑢}

18. Which of the following is true about the above information?

(A) 𝑀 ⊂ 𝑁

(B) 𝑀 ∩ 𝑁 = ∅

(C) 𝑀 ∩ 𝑁 = 𝑀

(D)𝑀 ∩ 𝑁 = 𝑁

19. 𝑀 ∩ 𝑁 ∩ 𝑃 =

(A) {∅}

(B) {𝑎, 𝑏}

(C) {𝑏, 𝑐}

(D){𝑎, 𝑏, 𝑐, 𝑒, 𝑖, 𝑜, 𝑢}

20. Which of the following would form disjoint sets?

(A) {𝑒𝑣𝑒𝑛 𝑛𝑢𝑚𝑏𝑒𝑟𝑠} and {𝑜𝑑𝑑 𝑛𝑢𝑚𝑏𝑒𝑟𝑠}

(B) {𝑝𝑟𝑖𝑚𝑒 𝑛𝑢𝑚𝑏𝑒𝑟𝑠} and {𝑟𝑎𝑡𝑖𝑜𝑛𝑎𝑙 𝑛𝑢𝑚𝑏𝑒𝑟𝑠}

(C) {𝑖𝑟𝑟𝑎𝑡𝑖𝑜𝑛𝑎𝑙 𝑛𝑢𝑚𝑏𝑒𝑟𝑠} and {𝑟𝑒𝑎𝑙 𝑛𝑢𝑚𝑏𝑒𝑟𝑠}

(D){𝑚𝑢𝑙𝑡𝑖𝑝𝑙𝑒𝑠 𝑜𝑓 3} and {𝑓𝑎𝑐𝑡𝑜𝑟𝑠 𝑜𝑓 27}

57
Answer Key:

1. D

2. B

3. C

4. C

5. C

6. C

7. B

8. D

9. C

10. C

11. C

12. D

13. C

14. C

15. D

16. C

17. C

18. D

19. A

20. A

58
Past Paper Question – June 2023 – Paper 2 – Question 1

1. (a) Find the EXACT value of

5 2 12 7
+ 3 − 35 × 9 [2]
6

(b) (i) Calculate the value of √1 − (cos 37°)2 correct to 3 decimal places. [2]

(ii) Write 0.00527 in standard form. [1]

(c) Haresh works at a call centre for 35 hours each week. He is paid an hourly rate

of $11.20.

(i) Calculate the amount of money Haresh earns in a four-week

month. [2]

(ii) In a certain week, Haresh works 8 hours overtime. Overtime hours are
1
paid at 1 2 times the usual rate of $11.20 per hour.

Find the TOTAL amount of money Haresh is paid for that week. [2]

Total: 9 marks

59
Past Paper Question – June 2023 – Paper 2 – Question 1 – Solution

1. (a) Find the EXACT value of

5 2 12 7
+ 3 − 35 × 9 [2]
6

4 1
5 2 12 7 5 4 12 7
+ 3 − 35 × 9 = (6 + 6) − 35 × 9
6 3
5
5 2 12 7 9 4
+ 3 − 35 × 9 = 6 − 15
6

5 2 12 7 3 4
+ 3 − 35 × 9 = 2 − 15
6

5 2 12 7 45−8
+ 3 − 35 × 9 =
6 30

5 2 12 7 37
+ 3 − 35 × 9 = 30
6

(b) (i) Calculate the value of √1 − (cos 37°)2 correct to 3 decimal places. [2]

With the use of a calculator,

√1 − (cos 37°)2 = 0.602 (to 3 decimal places)

(ii) Write 0.00527 in standard form. [1]

0.00527 = 5.27 × 10−3

60
(c) Haresh works at a call centre for 35 hours each week. He is paid an hourly rate

of $11.20.

(i) Calculate the amount of money Haresh earns in a four-week

month. [2]

In one week, Haresh earns = 35 × $11.20

In one week, Haresh earns = $392

In four weeks, Haresh earns = $392 × 4

In four weeks, Haresh earns = $1568

(ii) In a certain week, Haresh works 8 hours overtime. Overtime hours are
1
paid at 1 2 times the usual rate of $11.20 per hour.

Find the TOTAL amount of money Haresh is paid for that week. [2]

Overtime rate = 1.5 × Basic rate

Overtime rate = 1.5 × $11.20

Overtime rate = $16.80

Overtime wage = Overtime rate × Overtime hours

Overtime wage = $16.80 × 8

Overtime wage = $134.40

In one week, his basic wage = 35 × $11.20

In one week, his basic wage = $392

61
Therefore,

Total wage for that week = Basic wage + Overtime wage

Total wage for that week = $392 + $134.40

Total wage for that week = $526.40

Total: 9 marks

62
Section 4 - Measurement

Objective 1: Convert units of length, mass, area, volume, capacity.

[Refer to Section 1, Objective 8]

Quantity Unit Symbol


Length metre 𝑚
Mass gram 𝑔
Area squared metre 𝑚2
Volume cubic metre 𝑚3
Capacity litre 𝐿

Recall:

1 𝑚 = 100 𝑐𝑚

1 𝑘𝑚 = 1000 𝑚

Conversion between 𝒄𝒎 and 𝒌𝒎

÷ 𝟏𝟎𝟎 ÷ 𝟏𝟎𝟎𝟎

𝑐𝑚 𝑚 𝑘𝑚

× 𝟏𝟎𝟎 × 𝟏𝟎𝟎𝟎

1 litre = 1000 𝑐𝑚3

100 000 𝑐𝑚 = 1 𝑘𝑚

1 𝑚2 = 10 000 𝑐𝑚2

63
Objective 2: Use the appropriate SI unit of measure for area, volume, capacity, mass,

temperature and time (24-hour clock) and other derived quantities.

[Refer to Section 1, Objective 8]

The basic quantities are listed in the table below:

Quantity SI unit of measure Symbol

Area squared metre 𝑚2

Volume cubic metre 𝑚3

Capacity litre 𝐿

Mass kilogram 𝑘𝑔

Temperature degree Celsius °𝐶

Time seconds 𝑠

Note:

The 24-hour clock is stated in the form ℎℎ: 𝑚𝑚, where ℎℎ (00 to 23) is the number of full

hours that have passed since midnight and 𝑚𝑚 is the number of full minutes that have

passed. It uses the numbers 00:00 until 23:59 to tell the time.

Some derived quantities are listed in the table below:

Quantity SI unit of measure Symbol

Speed metre per second 𝑚𝑠 −1

Acceleration metre per second squared 𝑚𝑠 −2

64
Objective 3: Determine the perimeter of a plane shape.

Square

𝑠 𝑠

Perimeter of square = 𝑠 + 𝑠 + 𝑠 + 𝑠

Perimeter of square = 4𝑠

Rectangle

𝑤 or 𝑏

Perimeter of rectangle = 2𝑤 + 2𝑙

Perimeter of rectangle = 2(𝑤 + 𝑙)

65
Triangle

𝑏 𝑐

Perimeter of triangle = 𝑎 + 𝑏 + 𝑐

Trapezium

𝑎
V

V

V

Perimeter of trapezium = (add up all the sides)

Circle

𝑑𝑖𝑎𝑚𝑒𝑡𝑒𝑟

66
Diameter = 2 × radius

Note: 𝜋 is approximately
22
Circumference of a circle = 𝜋d equal to or 3.14.
7

= 2𝜋𝑟

Tip: Ensure to use the value of 𝜋 as stated in the question.

Compound Shapes

𝑳 𝑴

𝟒𝟗 𝒎

𝑵 𝟗𝟖 𝒎 𝑷

Tips:

1. When calculating the perimeter of this shape, do NOT include the lengths of 𝐿𝑁

and 𝑀𝑃. Perimeter means distance around the shape.

2. When substituting the value of 𝑟 into your equation for the circumference of the

circle, ensure you do NOT substitute the value of the diameter.


1
3. To find the circumference of a semicircle, it would be 2 × circumference of circle.

67
Objective 4: Calculate the length of an arc of a circle.

An arc of a circle is any portion of the circumference of a circle.

𝒎𝒊𝒏𝒐𝒓 𝒂𝒓𝒄

𝒎𝒂𝒋𝒐𝒓 𝒂𝒓𝒄

Formula:

𝜽
Length of an arc = 𝟑𝟔𝟎° 𝟐𝝅𝒓

VITAL: Ensure that your calculator is in DEGREES mode.

Tip:

Ensure to read your question properly if it asks for the length of the major arc or minor

arc.

68
Objective 5: Estimate the area of plane shapes.

Objective 6: Calculate the area of polygons and circles.

Square

𝑠 𝑠

Area of square = 𝑠 × 𝑠

Area of square = 𝑠 2

Rectangle

𝑤 or 𝑏

Area of rectangle = 𝑙 × 𝑤

Area of rectangle = 𝑙𝑤

69
Triangle

3 Formulas for Area of a Triangle


1
1. Area of triangle = 2 𝑏ℎ (where ℎ is perpendicular to the base)

2. Heron’s formula :

𝑎+𝑏+𝑐
Area = √𝑠(𝑠 − 𝑎)(𝑠 − 𝑏)(𝑠 − 𝑐) where 𝑠 = semi-perimeter = 2

1
3. Area of triangle = 2 𝑎𝑏 sin 𝐶

Trapezium

𝑎
V


V

1
Area of trapezium = 2 (𝑎 + 𝑏)ℎ

1
= (sum of parallel sides) × ℎ Note: The height is
2

perpendicular to the base.

(where ℎ is perpendicular to the base)


70
Circle

𝑑𝑖𝑎𝑚𝑒𝑡𝑒𝑟

Area of a circle = 𝜋𝑟 2

71
Objective 7: Calculate the area of a sector of a circle.

A sector is the portion of the circle enclosed by two radii and an arc, where the smaller

area is known as the minor sector and the larger being the major sector.

𝒎𝒊𝒏𝒐𝒓 𝒔𝒆𝒄𝒕𝒐𝒓

𝒎𝒂𝒋𝒐𝒓 𝒔𝒆𝒄𝒕𝒐𝒓

Formula:

𝜽
Area of a sector = 𝟑𝟔𝟎° 𝝅𝒓𝟐

VITAL: Ensure that your calculator is in DEGREES mode.

Tip:

Ensure to read your question properly if it asks for the area of the major sector or the

minor sector.

72
Objective 8: Calculate the area of a triangle given two sides and the angle they form.

Example:

45°
𝐶

Find the area of the triangle.

Solution:

1
Area of triangle = 2 𝑎𝑏 sin 𝐶

1
= 2 (10)(11) sin(45)

= 38.9 𝑐𝑚2 (to 3 significant figures)

Tips:

1. The angle used in the formula must be the included angle between the two

knows sides.

2. Write your answer to 3 significant figures unless otherwise stated.

3. Ensure to include the appropriate units in your final answer.

73
Objective 9: Calculate the area of a segment of a circle.

Consider the diagram below:

𝐴 𝐵

Formula: Area of the shaded region = Area of the sector – Area of the triangle

74
Objective 10: Calculate the surface area of solids.

Definition:

The surface area is defined as the combined area of all of the faces on a three-dimensional

object.

Cuboid

A cuboid is a solid which has six rectangular faces at right angles to each other.

Height

Width

Length

Net Diagram:

75
Formula:

Surface Area of cuboid = 𝟐(𝒘𝒍 + 𝒍𝒉 + 𝒘𝒉)

Cylinder

A cylinder is a solid geometrical figure with straight parallel sides and a circular or oval

cross section.

Net Diagram:


2𝜋𝑟

76
Formula:

Surface Area of cylinder = 𝟐𝝅𝒓𝟐 + 𝟐𝝅𝒓𝒉

Sphere

Formula:

Surface Area of a sphere = 𝟒𝝅𝒓𝟐

77
Objective 11: Calculate the volume of solids.

Formulas:

Volume of a cube = 𝑠 × 𝑠 × ℎ

Volume of a cuboid = 𝑙 × 𝑏 × ℎ

Volume of cylinder = 𝜋𝑟 2 ℎ

4
Volume of a sphere = 3 𝜋𝑟 3

1
Volume of cone = 𝜋𝑟 2 ℎ
3

A prism has a uniform cross-sectional area.

Volume of a prism = Cross Sectional Area × length

78
Objective 12: Solve problems involving the relations among time, distance and speed.

Quantity Unit Symbol for unit

Time second 𝑠

Distance metre 𝑚

Speed metre per second 𝑚/𝑠 or 𝑚𝑠 −1

Acceleration metre per second squared 𝑚/𝑠 2 or 𝑚𝑠 −2

Speed:

Speed is defined as the rate of change of distance. It is the distance moved per unit time.

𝐃𝐢𝐬𝐭𝐚𝐧𝐜𝐞
Speed = 𝐓𝐢𝐦𝐞

Another unit for speed is 𝑘𝑚/ℎ𝑟.

Conversion between 𝒌𝒎/𝒉𝒓 and 𝒎/𝒔

To convert 𝒌𝒎/𝒉𝒓 to 𝒎/𝒔:

𝟏𝟎𝟎𝟎
Multiply by 1000 and then divide by 3600 OR × 𝟑𝟔𝟎𝟎

To convert 𝒎/𝒔 to 𝒌𝒎/𝒉𝒓:

𝟑𝟔𝟎𝟎
Divide by 1000 and then multiply by 3600 OR × 𝟏𝟎𝟎𝟎

Acceleration:

Acceleration is defined as the rate of change of speed.

𝐒𝐩𝐞𝐞𝐝
Acceleration = 𝐓𝐢𝐦𝐞

The unit for acceleration is 𝑚𝑠 −2 .

79
You can be tested on two types of graphs:

1. Distance-Time graphs

2. Speed-Time graphs

Here are some important points you should be aware of:

Distance vs. Time Speed vs. Time

Gradient speed acceleration

zero speed constant speed


(at rest)

▪ distance increasing ▪ speed increasing

▪ constant speed ▪ constant acceleration

▪ (forward direction)

▪ distance increasing ▪ speed decreasing

in opposite direction ▪ constant deceleration

▪ constant speed

▪ (reverse direction)

Note:

In a speed vs. time graph, if you work out the AREA under the graph, you will find the

DISTANCE.

Tip:

CXC loves trapeziums. Do ensure to know the formula to find the area of a trapezium if

asked to find the area under the graph.


1
Area of trapezium = 2 (𝑎 + 𝑏)ℎ

80
Objective 13: Estimate the margin of error for a given measurement.

Definition:

The margin of error measures the range of values within which the true value of a

measurement is likely to fall.

For example, 𝑥 = 30 ± 1 𝑐𝑚 has a margin of error of 1. Hence, the measurement can fall

within the range 29 ≤ 𝑥 ≤ 31.

81
Objective 14: Use scales and scale drawings to determine distances and areas

Example:

Scale is 1:50 000

𝑷 𝑸

(a) What is the actual distance from 𝑃 to 𝑄?

(b) What is the area in 𝑐𝑚2 represented on the map of the island?

Solution:

(a) On the map, it is 4 𝑐𝑚 from 𝑃 to 𝑄.

In reality, from 𝑃 to 𝑄 = 50 000 × 4

In reality, from 𝑃 to 𝑄 = 200 000 𝑐𝑚

(b) By counting squares, the area = 12 𝑐𝑚2

Tips:

1. Count the whole number of squares first.

2. Then, count the pieces such that they add up to a whole square.

82
Objective 15: Solve problems involving measurement.

Tips:

1. Ensure to state any formula used in your solution.

2. Include units in your final answers. If you are not given the units, simply write

units.

3. Use the value of 𝜋 stated in the question.

4. Read your question properly to know if they asked you to place your numbers in

a particular way such as 2 decimal places or 3 significant figures.

5. Work with the entire length of the number throughout the solution. Only when

you reach your final answer, then you can approximate accordingly.

83
Past Paper Question – January 2019 – Paper 2 – Question 6

The diagram below, not drawn to scale, shows an open cylindrical container made of

metal with a circular base and uniform thickness throughout. The length of the container,

from the top to outer bottom, is 120 cm and the inner and outer radii are 14 cm and 15

cm respectively.

𝟐𝟐
Take 𝝅 to be .
𝟕

(a) Draw a cross-sectional view of the container showing the measurements of the

inner and outer radii. [2]

(b) Show that the capacity of the container is 73 304 𝑐𝑚3 . [2]

(c) Determine the volume of the material used to make the container. [3]

(d) Given that the density of the material used to make the container is 2.2 g/cm3 ,

determine the mass, in kg, of the empty container.

𝐦𝐚𝐬𝐬
(𝐝𝐞𝐧𝐬𝐢𝐭𝐲 = 𝐯𝐨𝐥𝐮𝐦𝐞) [2]

Total: 9 marks

84
Past Paper Question – January 2019 – Paper 2 – Question 6 – Solution

The diagram below, not drawn to scale, shows an open cylindrical container made of

metal with a circular base and uniform thickness throughout. The length of the container,

from the top to outer bottom, is 120 cm and the inner and outer radii are 14 cm and 15

cm respectively.

𝟐𝟐
Take 𝝅 to be .
𝟕

(a) Draw a cross-sectional view of the container showing the measurements of the

inner and outer radii. [2]

1 𝑐𝑚

85
(b) Show that the capacity of the container is 73 304 𝑐𝑚3 . [2]

Volume = 𝜋𝑟 2 ℎ
22
Volume = × (14)2 × 119
7

Volume = 73 304 𝑐𝑚3

Hence, the capacity of the container is 73 304 𝑐𝑚3 .

(c) Determine the volume of the material used to make the container. [3]

External volume of the cylinder = 𝜋𝑟 2 ℎ


22
External volume of the cylinder = × (15)2 × 120
7

External volume of the cylinder = 84 857.14 𝑐𝑚3

= Volume of the material used

= External volume of the container – Internal volume of the container

= 84 857.14 − 73 304

= 11 553.14 𝑐𝑚3 (to 2 decimal places)

86
(d) Given that the density of the material used to make the container is 2.2 g/cm3 ,

determine the mass, in kg, of the empty container.

𝐦𝐚𝐬𝐬
(𝐝𝐞𝐧𝐬𝐢𝐭𝐲 = 𝐯𝐨𝐥𝐮𝐦𝐞) [2]

𝑀𝑎𝑠𝑠
Density = 𝑉𝑜𝑙𝑢𝑚𝑒

∴ Mass = Density × Volume

∴ Mass = 2.2 × 11 553.14

∴ Mass = 25 416.91 𝑔

Now,

1000 𝑔 = 1 𝑘𝑔
1
1 𝑔 = 1000

1
25 416.91 𝑔 = × 25 416.91
1000

25 416.91 𝑔 = 25.4 𝑘𝑔 (to 2 decimal places)

Total: 9 marks

87
Past Paper Question – June 2023 – Paper 2 – Question 6
22
6. [In this question, take 𝜋 = and the volume, 𝑉, of a cone with radius 𝑟 and height
7

1
ℎ as 𝑉 = 3 𝜋𝑟 2 ℎ.]

The diagram below shows a sector 𝑂𝑀𝑅𝑁, of a circle with centre 𝑂, radius 12 𝑐𝑚

and sector angle 168°, which was formed using a thin sheet of metal.

𝟏𝟔𝟖°
𝑴
𝑵

(a) Calculate the perimeter of the sector above, made from the thin sheet of

metal. [3]

(b) A cone is made from the sector in (a) by joining 𝑂𝑀 to 𝑂𝑁, as shown below.

𝒓 𝑴
𝑵
88
(i) Calculate the

(a) radius, 𝑟, of the cone [2]

(b) height, ℎ, of the cone [2]

(ii) Calculate the capacity of the cone, in litres. [2]

Total: 9 marks

89
Past Paper Question – June 2023 – Paper 2 – Question 6 – Solution
22
6. [In this question, take 𝜋 = and the volume, 𝑉, of a cone with radius 𝑟 and height
7

1
ℎ as 𝑉 = 3 𝜋𝑟 2 ℎ.]

The diagram below shows a sector 𝑂𝑀𝑅𝑁, of a circle with centre 𝑂, radius 12 𝑐𝑚

and sector angle 168°, which was formed using a thin sheet of metal.

𝟏𝟔𝟖°
𝑴
𝑵

(a) Calculate the perimeter of the sector above, made from the thin sheet of

metal. [3]

𝜃
Length of arc 𝑀𝑅𝑁 = 360° × 2𝜋𝑟

168° 22
Length of arc 𝑀𝑅𝑁 = 360° × 2 × × 12
7

Length of arc 𝑀𝑅𝑁 = 35.2 𝑐𝑚

Perimeter of sector = 𝑂𝑀 + 𝑂𝑁 + 𝑀𝑅𝑁

Perimeter of sector = 12 + 12 + 35.2

Perimeter of sector = 59.2 𝑐𝑚

90
(b) A cone is made from the sector in (a) by joining 𝑂𝑀 to 𝑂𝑁, as shown below.

𝒓 𝑴
𝑵

(i) Calculate the

(a) radius, 𝑟, of the cone [2]

From the diagram, the circumference of the base of the cone = Arc

length 𝑀𝑁

∴ 2𝜋𝑟 = 35.2

35.2
𝑟= 2𝜋

35.2
𝑟= 22
2( )
7

𝑟 = 5.6 𝑐𝑚

∴ Radius of the cone, 𝑟 = 5.6 𝑐𝑚

(b) height, ℎ, of the cone [2]

Consider the triangle below:

91
𝒉 𝟏𝟐 𝒄𝒎

𝒓 = 𝟓. 𝟔 𝒄𝒎

By Pythagoras’ Theorem,

𝑎2 + 𝑏 2 = 𝑐 2

ℎ2 + (5.6)2 = (12)2

ℎ2 = (12)2 − (5.6)2

ℎ2 = 144 − 31.36

ℎ2 = 112.64

ℎ = √112.64

ℎ = 10.6 𝑐𝑚 (to 3 significant figures)

∴ The height, ℎ, of the cone is 10.6 𝑐𝑚.

(ii) Calculate the capacity of the cone, in litres. [2]

1
Volume of the cone = 3 𝜋𝑟 2 ℎ

1 22
Volume of the cone = 3 × × (5.6)2 × 10.6
7

Volume of the cone = 348.2 𝑐𝑚3 (to 1 decimal place)

92
Now,

1000 𝑐𝑚3 = 1 litre


1
1 𝑐𝑚3 = 1000 litre

1
348.2 𝑐𝑚3 = 1000 × 348.2

348.2 𝑐𝑚3 = 0.348 𝑙 (to 3 significant figures)

∴ The capacity of the cone is 0.348 𝑙.

Total: 9 marks

93
Section 5 - Statistics

Objective 1: Differentiate between sample and population attributes.

A population is the entire group you want to draw conclusions about.

A sample is the specific group that you will collect data from.

94
Objective 2: Construct a frequency table for a given set of data.

You should know the difference between discrete and continuous variables as well as

ungrouped and grouped data.

Discrete Variables

➢ Count of individual items or values

➢ Takes set values between an interval

Example: number of red balls in a bag

CSEC Definition:

Discrete – A set of values are said to be discrete if they are all distinct and separated from

each other. For example, the set of shoe sizes where the elements can only take on a

limited and distinct set of values.

Continuous Variables

➢ Measurements of continuous values

➢ Has infinite values between an interval

Example: height of students in a class

95
Ungrouped Data

Ungrouped data is the data you first gather from an experiment or study. The data is raw,

that is, it is not sorted into categories, classified, or otherwise grouped.

Example: Marbles in a bag

red 6

blue 3

white 5

Grouped Data

Grouped data is data that has been organised into categories or classified into groups.

Example of grouped data:

Age Intervals Tally Frequency


11-15 ||| 3
16-20 |||| 5
21-25 |||| ||| 8
26-30 | 1

96
Constructing a Frequency Table:

Example:

A survey was conducted and the ages of 20 people were recorded. The raw data collected

is as follows:

3 , 2, 21 , 20 , 16

15 , 14 , 3, 11 , 10
Note: This is ungrouped data.
1 , 19 , 24 , 17 , 12

7 , 8, 4, 2, 3

The following table shows the grouped data of the age of 20 people:

Age Intervals Tally Frequency


1-5 |||| || 7
6-10 ||| 3
11-15 |||| 4
16-20 |||| 4
21-25 || 2

Tips:

1. When completing the Tally column, use the “Crossing Out Method”. Begin filling in

the Tally column by crossing out one number at a time from the raw data. This

helps to reduce errors.

2. After completing the table, double check that the sum of the frequency (∑ 𝑓) is

equal to the number of items in the data.

97
Objective 3: Determine class features for a given set of data.

Class Interval

➢ describes the class using its limits

➢ name/handle

CSEC Definition:

Class Interval – Non-overlapping intervals, which together contain every piece of data in

a survey.

Question: How many persons are in the class interval 11-15?

Answer: 4 persons

Class Limits

➢ the class limits may be defined as the minimum value and the maximum value

the class interval may contain.

➢ given number for the interval.

Question: What are the limits for the second class interval?

Answer: 6 and 10

Question: What is the upper class limit of the third class interval?

Answer: 15

98
Class Boundary

➢ actual border for values in the class interval.

Question: What is the lower class boundary for the second class interval?

Answer: 5.5

Question: What is the upper class boundary for the second class interval?

Answer: 10.5

Class Width

➢ the difference between the upper class boundary and the lower class boundary.

Formula:

Class Width = Upper class boundary − Lower class boundary

Class Width = UCB − LCB

Question: What is the class width for the grouped data of the age of 20 people?

Answer:

Class width = UCB – LCB OR Class width = UCB – LCB

= 5.5 − 0.5 = 10.5 − 5.5

=5 =5

99
Class Midpoint

➢ the number in the middle of the class. It is found by adding the upper and lower

limits and dividing by two.

➢ It can also be found by adding the upper and lower boundaries and dividing by

two.

Formula:

There are two formulas that can be used to calculate the class midpoint:

𝐔𝐩𝐩𝐞𝐫 𝐂𝐥𝐚𝐬𝐬 𝐋𝐢𝐦𝐢𝐭 + 𝐋𝐨𝐰𝐞𝐫 𝐂𝐥𝐚𝐬𝐬 𝐋𝐢𝐦𝐢𝐭


Class Midpoint = 𝟐

𝐔𝐩𝐩𝐞𝐫 𝐂𝐥𝐚𝐬𝐬 𝐁𝐨𝐮𝐧𝐝𝐚𝐫𝐲 + 𝐋𝐨𝐰𝐞𝐫 𝐂𝐥𝐚𝐬𝐬 𝐁𝐨𝐮𝐧𝐝𝐚𝐫𝐲


Class Midpoint = 𝟐

Question: What is the class midpoint for the first class interval?

Answer:

Upper Class Limit + Lower Class Limit


Class Midpoint = 2

5+1
= 2

6
=2

=3

100
Objective 4: Construct statistical diagrams.

Pie Chart

A pie chart is a chart that uses pie slices to show relative sizes of data.

Note:

When drawing a pie chart, ensure to:

- Use a ruler

- Put a title

- Insert the labels in the chart

- Include the angle measurements

You are NOT required to shade any of the sectors.

Title: Pie chart showing the sports played by students

cricket

165°
120° football
75°

basketball

101
Advantages of using a Pie Chart

➢ A pie chart provides a visual representation of each category and its proportion.

➢ You can get an idea of the fraction of each share with one glance.

102
Bar Graph

A bar graph represents categorical data with rectangular bars with heights proportional

to the values that they represent.

Characteristics of a bar graph:

• Evenly spaced columns

• Uses discrete data

• Height of column represents the frequency

• Widths of the column are the same

• Bars can be horizontal or vertical

Title: Bar Graph showing the number of accidents from 1999-2002


Frequency /thousands of accidents

Year

103
Tips:

1. Take note of the unit of the 𝑦-axis. It is in thousands of accidents. Therefore,

when stating the number of accidents in 1999, it will be 10 000 accidents, not 10

accidents.

2. If asked to state between which two years was there the greatest increase in the

number of accidents, the answer would be 1999 to 2000. Some students

mistakenly write 2000 which is simply the year that had the most accidents.

104
Line Graph

A line graph is a type of chart used to visualize the value of something over time.

Example:

The table below shows the number of cars manufactured from 1998-2001.

Year Number of cars manufactured


1998 10 000
1999 20 000
2000 15 000
2001 17 000

Title: Line Graph showing the number of cars manufactured from 1998-2001.

Scale
𝑥-axis: 1 cm = 1 year
𝑦-axis: 1 cm = 5 000 cars
Frequency (1000’s of cars)

Years

105
Histogram

A histogram displays the shape and spread of continuous sample data. The columns of a

histogram “touch” each other.

Example:

The table below shows the grouped data of the age of 29 persons.

Age Intervals Frequency

1-10 3

11-20 5

21-30 12

31-40 7

41-50 2

Draw a histogram to represent the information in the table above.

Note:

When drawing a histogram, here are some useful tips:

- First, determine the upper class boundary and lower class boundary for each

class interval of the grouped data given.

- Draw the 𝑥-axis and 𝑦-axis using an appropriate scale.

- On the graph, insert a small dot at the UCB and LCB values for each interval

based on the frequency so you would be able to easily draw the height of the

columns.

- Draw the vertical lines first and then the horizontal lines.

106
Age Intervals Frequency Lower class Boundary Upper class Boundary

1-10 3 0.5 10.5

11-20 5 10.5 20.5

21-30 12 20.5 30.5

31-40 7 30.5 40.5

41-50 2 40.5 50.5

Title: Histogram showing the age of 29 persons

Scale
𝑥-axis: 1 cm = 5 years
𝑦-axis: 1 cm = 1 person

107
Frequency Polygon

A frequency polygon is a graph constructed by using lines to join the midpoints of each

interval and can be created from the histogram.

Age Intervals Frequency Midpoint

1-10 3 5.5

11-20 5 15.5 Note: In your frequency


polygon graph, ensure
21-30 12 25.5
to “close” the polygon.
31-40 7 35.5

41-50 2 45.5

Title: Frequency Polygon showing the age of 29 persons

Scale
𝑥-axis: 1 cm = 10 years
𝑦-axis: 1 cm = 2 persons
Frequency

Age (years)

108
Objective 5: Determine measures of central tendency for raw, ungrouped and grouped

data.

Objective 6: Determine when it is most appropriate to use the mean, median and mode

as the average for a set of data.

Mode

The mode of a set of numbers is the number that occurs most frequently.

Median

The median is the “middle” of a sorted list of numbers.

Mean

The mean of a set of figures is the sum of the figures divided by the number of figures in

the set.

Ungrouped Data: Mode, Median, Mean

Consider the following raw data collected:

2 , 1 , 0 , 3 , 4 , 6 , 2 , 0 , 3 , 3 , 5

Find:

(i) Mode

(ii) Mean

(iii) Median

109
Solution:

(i) Mode = 3 (as it occurs the most frequently)

𝑆𝑢𝑚 𝑜𝑓 𝑎𝑙𝑙 𝑑𝑎𝑡𝑎


(ii) Mean = 𝐹𝑟𝑒𝑞𝑢𝑒𝑛𝑐𝑦

29
= 11

= 2.6

(iii) Tip: Before finding the median, ensure that you put the list of numbers in

order.

0 , 0 , 1 , 2 , 2 , 3 , 3 , 3 , 4 , 5 , 6

Median = 3

Grouped Data: Mode, Median, Mean

The following table shows the grouped data of test scores out of 50:

Test Scores Intervals Frequency

1-10 1

11-20 3

21-30 3

31-40 7

41-50 6

Find:

110
(i) Modal Interval

(ii) Mean

(iii) Median

Solution:

(i) The modal interval is 31-40. (as it occurs the most)

(ii) Required to find the mean

Test Scores Intervals Frequency Midpoint, 𝒙 𝒇𝒙

1-10 1 5.5 5.5

11-20 3 15.5 46.5

21-30 3 25.5 76.5

31-40 7 35.5 248.5

41-50 6 45.5 273.0

∑𝒇 = 𝟐𝟎 ∑𝒇𝒙 = 𝟔𝟓𝟎

∑𝒇𝒙
Mean = ∑𝒇

650
= 20

= 32.5 is the test score

(iii) Median Interval: 31-40

Median = 35.5

Note: Looking at the “Frequency” column, you would be able to deduce that
the median will lie in the interval 31-40. You can also add the “Tally” column
and eliminate values on both ends to get the median value.
111
Objective 7: Determine the measures of dispersion (spread) for raw, ungrouped and

grouped data.

You should be able to estimate the following measures for grouped data:

1. Range

2. Interquartile range

3. Semi-interquartile range

Range

The range of a set of data is the difference between the highest and lowest values of the

data.

Formula:

Range = Highest Value – Lowest Value

Interquartile range

The IQR (interquartile range) describes the middle 50% of values when ordered from

highest to lowest. It is a measure of the dispersion of data.

Formula:

IQR = 𝑸𝟑 − 𝑸𝟏

where IQR = interquartile range

𝑄3 = 3rd quartile

𝑄1 = 1st quartile

112
𝑛+1
𝑄1, also known as the lower quartile or first quartile occurs at the th value.
4

𝑛+1
𝑄2 , also known as the median occurs at the th value.
2

3(𝑛+1)
𝑄3 , also known as the upper quartile or third quartile occurs at the th value.
4

Semi-interquartile range

The semi-interquartile range is half of the interquartile range.

Formula:

𝐈𝐐𝐑
SIQR = 𝟐

Example using grouped data:

Cumulative
Frequency Scale
𝑥-axis: 1 cm = 5 years
𝑦-axis: 1 cm = 10 persons

Age (years)
113
Question: Determine the median of the cumulative frequency curve below.

Solution:

𝑛+1 60+1 61
The median value occurs at the = = = 30.5th value
2 2 2

OR

𝑛 60
The median value occurs at the 2 = = 30th value
2

Using the cumulative frequency curve, the median age is 20 years.

Question: Determine the value of 𝑄3 .

Solution:

3(𝑛+1) 3(60+1)
The 3rd quartile occurs at the = = 45.75th value
4 4

Using the cumulative frequency curve, the value of 𝑄3 = 23 years.

Question: Determine the value of 𝑄1.

Solution:

𝑛+1 60+1
The 1st quartile occurs at the = = 15.25th value
4 4

Using the cumulative frequency curve, the value of 𝑄1 = 17 years.

114
Example using raw data:

The data below shows the ages of girls/women who received flowers on Valentine’s Day

from a particular street.

Raw Data: 6, 13, 15, 41, 52, 60, 66

Find

(a) 𝑄1

(b) 𝑄2

(c) 𝑄3

Solution:

We first need to ensure that the data is listed in ascending order.

6 13 15 41 52 60 66

The total number of observations in the data set, 𝑛 = 7.

1
(a) 𝑄1 = 4 (𝑛 + 1)

1
𝑄1 = 4 (7 + 1)

1
𝑄1 = 4 (8)

𝑄1 = 2nd term

∴ The value of 𝑄1 = 13

115
1
(b) 𝑄2 = 2 (𝑛 + 1)

1
𝑄2 = 2 (7 + 1)

1
𝑄2 = 2 (8)

𝑄2 = 4th term

∴ The value of 𝑄2 = 41

3
(c) 𝑄3 = 4 (𝑛 + 1)

3
𝑄3 = 4 (7 + 1)

3
𝑄3 = 4 (8)

𝑄3 = 6th term

∴ The value of 𝑄3 = 60

116
Objective 8: Use standard deviation to compare sets of data.

No calculation of the standard deviation will be required.

Note:

When comparing the standard deviation of a set of values, vital words to include are:

- “spread”
- “deviates”
- “mean”

Example:

Diagram 1:

Standard deviation = 5

117
Diagram 2:

Standard deviation = 1

For diagram 1, suppose that the standard deviation is 5 and for diagram 2, suppose that

the standard deviation is 1. The standard deviation of diagram 1 is larger than the

standard deviation of diagram 2. Since standard deviation is a measure of the spread of

data, it means that the values of diagram 1 were more ‘spread out’ or distributed over its

range than the values in diagram 2. In diagram 1, they deviated further from the mean

than in diagram 2.

118
Objective 9: Draw cumulative frequency curve (ogive).

➢ uses continuous data

➢ the ‘running total’ of frequencies

➢ the sum of the class and all classes below it in a frequency distribution.

Note:

When drawing a cumulative frequency curve, here are some useful tips:

- First, determine the upper class boundary for each class interval of the grouped

data given as these values will be used to plot the cumulative frequency curve.

- In this case, the 𝑥-axis will represent age and 𝑦-axis is almost always used to

represent the cumulative frequency.

- The shape of the curve looks like a ‘stretched 𝑆’ and is called an ogive.

- Use a small ‘×’ to plot the points and a fine pencil to connect them.

- Try to use as much of the graph paper as possible.

Example:

Age Intervals Frequency Cumulative Frequency Upper Class Boundary

1-10 2 2 10.5

11-20 9 11 20.5

21-30 25 36 30.5

31-40 30 66 40.5

41-50 10 76 50.5

51-60 4 80 60.5

119
Title: Cumulative Frequency Graph showing the age of 80 persons

Cumulative
Frequency

Scale
𝑥-axis: 1 cm = 5 years
𝑦-axis: 1 cm = 5 persons

Age (years)

120
Objective 10: Analyse statistical diagrams.

You need to be able to find the mean, mode, median, range, quartiles, interquartile range,

semi-interquartile range, trends and patterns.

121
Objective 11: Determine the proportion or percentage of the sample above or below a

given value from raw data, frequency table or cumulative frequency curve.

Formula:

𝑵𝒖𝒎𝒃𝒆𝒓 𝒐𝒇 𝑫𝒆𝒔𝒊𝒓𝒆𝒅 𝑶𝒖𝒕𝒄𝒐𝒎𝒆𝒔


Probability = 𝑻𝒐𝒕𝒂𝒍 𝒏𝒖𝒎𝒃𝒆𝒓 𝒐𝒇 𝒐𝒖𝒕𝒄𝒐𝒎𝒆𝒔

Example:

Using the cumulative frequency above, what is the probability of selecting a person over

50 years old?

Solution:

Using the cumulative frequency curve, it can be deduced that 76 persons are under the

age of 50.

Hence, 80 − 76 = 4 persons are over the age of 50.

𝑁𝑢𝑚𝑏𝑒𝑟 𝑜𝑓 𝐷𝑒𝑠𝑖𝑟𝑒𝑑 𝑂𝑢𝑡𝑐𝑜𝑚𝑒𝑠


Probability = 𝑇𝑜𝑡𝑎𝑙 𝑛𝑢𝑚𝑏𝑒𝑟 𝑜𝑓 𝑜𝑢𝑡𝑐𝑜𝑚𝑒𝑠

4
= 80

1
= 20 or 0.05 or 5%

122
Objective 12: Identify the sample space for simple experiment.

Objective 13: Determine experimental and theoretical probabilities of simple events.

Objective 14: Make inference(s) from statistics.

Coin

A coin has two possible outcomes, heads or tails.

The sample space is 𝑆 = {𝐻, 𝑇}.

Dice

The sample space for rolling a fair six-sided dice consists of six possible outcomes, each

corresponding to the number on the top face of the dice after it has been rolled.

The sample space is 𝑆 = {1, 2, 3, 4, 5, 6}.

Playing cards

The sample space for playing cards consists of 52 possible outcomes, each corresponding

to a different card in a standard deck of 52 playing cards.

𝑆 = {A♣, 2♣, 3♣, . . . , K♣, A♦, 2♦, 3♦, . . . , K♦, A♥, 2♥, 3♥, . . . , K♥, A♠, 2♠, 3♠, . . . , K♠}

where S is the sample space, and each element of the set represents a possible outcome

of drawing a single card from a deck of 52 playing cards.

Note that the deck contains four suits (clubs ♣, diamonds ♦, hearts ♥, and spades ♠), and

each suit contains 13 cards (Ace, 2, 3, 4, 5, 6, 7, 8, 9, 10, Jack, Queen, and King).

123
Use of Contingency Tables

Example:

Dog Cat Total

Male 42 10 52

Female 9 39 48

Total 51 49 100

(a) What is the probability of selecting a female at random?

(b) What is the probability of randomly selecting a male that prefers cats?

(c) Given that a female is selected, what is the probability that she likes dogs?

Solution:

𝑁𝑢𝑚𝑏𝑒𝑟 𝑜𝑓 𝑑𝑒𝑠𝑖𝑟𝑒𝑑 𝑜𝑢𝑡𝑐𝑜𝑚𝑒𝑠


(a) Probability = 𝑇𝑜𝑡𝑎𝑙 𝑛𝑢𝑚𝑏𝑒𝑟 𝑜𝑓 𝑜𝑢𝑡𝑐𝑜𝑚𝑒𝑠

48
Probability = 100

12
Probability = 25

𝑁𝑢𝑚𝑏𝑒𝑟 𝑜𝑓 𝑑𝑒𝑠𝑖𝑟𝑒𝑑 𝑜𝑢𝑡𝑐𝑜𝑚𝑒𝑠


(b) Probability = 𝑇𝑜𝑡𝑎𝑙 𝑛𝑢𝑚𝑏𝑒𝑟 𝑜𝑓 𝑜𝑢𝑡𝑐𝑜𝑚𝑒𝑠

10
Probability = 100

1
Probability = 10

𝑁𝑢𝑚𝑏𝑒𝑟 𝑜𝑓 𝑑𝑒𝑠𝑖𝑟𝑒𝑑 𝑜𝑢𝑡𝑐𝑜𝑚𝑒𝑠


(c) Probability = 𝑇𝑜𝑡𝑎𝑙 𝑛𝑢𝑚𝑏𝑒𝑟 𝑜𝑓 𝑜𝑢𝑡𝑐𝑜𝑚𝑒𝑠

9
Probability = 48

124
Past Paper Question – January 2009 – Paper 2 – Question 7

The table below shows the distribution of marks on a test for a group of 70 students.

Mark Frequency Cumulative Frequency


1 − 10 2 2
11 − 20 5 7
21 − 30 9 16
31 − 40 14
41 − 50 16
51 − 60 12
61 − 70 8
71 − 80 4 70

(a) Copy and complete the table to show the cumulative frequency for the

distribution. [2]

(b) (i) Using a scale of 1 𝑐𝑚 to represent 5 marks on the horizontal axis and 1 𝑐𝑚 to

represent 5 students on the vertical axis, draw the cumulative frequency curve
for the scores. [5]

(ii) What assumption have you made in drawing your curve through the point

(0, 0)? [1]

(c) The pass mark for the test is 47. Use your graph to determine the number of

students who passed the test. [2]

(d) What is the probability that a student chosen at random had a mark less than or

equal to 30? [2]

Total: 12 marks

125
Past Paper Question – January 2009 – Paper 2 – Question 7 – Solution

The table below shows the distribution of marks on a test for a group of 70 students.

Mark Frequency Cumulative Frequency


1 − 10 2 2
11 − 20 5 7
21 − 30 9 16
31 − 40 14 30
41 − 50 16 46
51 − 60 12 58
61 − 70 8 66
71 − 80 4 70

(a) Copy and complete the table to show the cumulative frequency for the

distribution. [2]

(b) (i) Using a scale of 1 𝑐𝑚 to represent 5 marks on the horizontal axis and 1 𝑐𝑚 to

represent 5 students on the vertical axis, draw the cumulative frequency curve
for the scores. [5]

Mark Frequency Cumulative Frequency Upper Class Boundary


1 − 10 2 2 10.5
11 − 20 5 7 20.5
21 − 30 9 16 30.5
31 − 40 14 30 40.5
41 − 50 16 46 50.5
51 − 60 12 58 60.5
61 − 70 8 66 70.5
71 − 80 4 70 80.5

126
Title: Cumulative Frequency Graph showing the distribution of marks on a test
for a group of 70 students.

Cumulative
Frequency

Scale
𝑥-axis: 1 cm = 5 marks
𝑦-axis: 1 cm = 5 students

Mark

(ii) What assumption have you made in drawing your curve through the point

(0, 0)? [1]

Using the starting point as (0, 0), you have assumed that no student obtained a

mark of 0.

127
(c) The pass mark for the test is 47. Use your graph to determine the number of

students who passed the test. [2]

From the graph, when the pass mark is 47, the cumulative frequency is 40.

Therefore,

Number of students that passed the test = 70 − 40

Number of students that passed the test = 30 students

(d) What is the probability that a student chosen at random had a mark less than or

equal to 30? [2]

From the graph, when 𝑥 = 30, 𝑦 = 15

The number of students who obtained a mark ≤ 30.

Therefore,

𝑁𝑢𝑚𝑏𝑒𝑟 𝑜𝑓 𝑑𝑒𝑠𝑖𝑟𝑒𝑑 𝑜𝑢𝑡𝑐𝑜𝑚𝑒𝑠


Probability =
𝑇𝑜𝑡𝑎𝑙 𝑛𝑢𝑚𝑏𝑒𝑟 𝑜𝑓 𝑜𝑢𝑡𝑐𝑜𝑚𝑒𝑠

15
Probability = 70

3
Probability = 14

Total: 12 marks

128
Past Paper Question – June 2023 – Paper 2 – Question 5

5. Each of 75 girls recorded the name of her favourite sport. The number of girls who

chose track and cricket are shown on the bar chart below.
Number of girls

Swimming Tennis Track Cricket Football

Favourite sport

(a) How many more girls chose cricket than track as their favourite sport? [1]

(b) Eleven girls recorded tennis as their favourite sports. For the remaining girls,

the number who chose swimming compared to the number who chose football

was in the ratio 2: 3.

Use this information to complete the bar chart above. [3]

129
(c) Determine the modal sport. [1]

(d) One of the girls is selected at random. What is the probability that she chose

NEITHER track NOR cricket as her favourite sport? [2]

(e) The information on the favourite sport of the 75 girls is to be shown on a pie

chart. Calculate the sector angle for football. [2]

Total: 9 marks

130
Past Paper Question – June 2023 – Paper 2 – Question 5 – Solution

5. Each of 75 girls recorded the name of her favourite sport. The number of girls who

chose track and cricket are shown on the bar chart below.
Number of girls

Swimming Tennis Track Cricket Football

Favourite sport

(a) How many more girls chose cricket than track as their favourite sport? [1]

Number of more girls who chose cricket than track = 17 − 12

Number of more girls who chose cricket than track = 5 girls

131
(b) Eleven girls recorded tennis as their favourite sports. For the remaining girls,

the number who chose swimming compared to the number who chose football

was in the ratio 2: 3.

Use this information to complete the bar chart above. [3]

Number of remaining girls = 75 − (12 + 17 + 11)

Number of remaining girls = 75 − 40

Number of remaining girls = 35

Swimming : Football

2:3

2
Number of girls who chose swimming = × 35
5

Number of girls who chose swimming = 14 girls

3
Number of girls who chose football = 5 × 35

Number of girls who chose football = 21 girls

(c) Determine the modal sport. [1]

The modal sport is Football.

132
(d) One of the girls is selected at random. What is the probability that she chose

NEITHER track NOR cricket as her favourite sport? [2]

Number of girls who chose track or cricket = 12 + 17

Number of girls who chose track or cricket = 29 girls

Number of girls who chose neither track nor cricket = 75 − 29

Number of girls who chose neither track nor cricket = 46 girls

𝑁𝑢𝑚𝑏𝑒𝑟 𝑜𝑓 𝑑𝑒𝑠𝑖𝑟𝑒𝑑 𝑜𝑢𝑡𝑐𝑜𝑚𝑒𝑠


Probability that she chose neither track nor cricket = 𝑇𝑜𝑡𝑎𝑙 𝑛𝑢𝑚𝑏𝑒𝑟 𝑜𝑓 𝑜𝑢𝑡𝑐𝑜𝑚𝑒𝑠

46
Probability that she chose neither track nor cricket = 75

(e) The information on the favourite sport of the 75 girls is to be shown on a pie

chart. Calculate the sector angle for football. [2]

21
Sector angle for football = 75 × 360°

504°
Sector angle for football = 5

Sector angle for football = 100.8°

Total: 9 marks

133
Section 6 - Algebra

Objective 1: Use symbols to represent numbers, operations, variables and relations.

• Numerical symbols are 0, 1, 2, 3, and so on represent specific quantities.

• You should be able to use algebraic symbols such as +, -, ×, ÷, =, and parentheses

to write algebraic expressions and equations that involve variables.

• You should be able to use variable symbols such as 𝑥, 𝑦, 𝑧 to represent unknown

quantities or quantities that can vary.

• You should be able to use inequality symbols to compare quantities. They are:

< less than

> greater than

≤ less than or equal to

≥ greater than or equal to

134
Objective 2: Translate between algebraic symbols and worded expressions.

Worded Statement: Four is subtracted from the product of 𝑥 and 𝑦.

Algebraic expression: 𝑥𝑦 − 4

Worded Statement: The ingredients for each small pizza cost $15 and for each large pizza

$30. The manager of a pizza shop wishes to make 𝑥 small pizzas and 𝑦 large pizzas. He

plans to spend no more than $450 on ingredients.

Algebraic inequality: 15𝑥 + 30𝑦 ≤ 450

135
Objective 3: Evaluate arithmetic operations involving directed numbers.

You should know the rules involving directed numbers.

Rule #1: For Multiplication, Division or if two Signs are next to each other

1st Sign 2nd Sign Result

+ + +

− − +

+ − −

− + −

Rule #2: For Addition and Subtraction

• Check to see if signs are next to each other. (If so, change to one sign)

• Same signs? → add the numbers → put back the sign

(without the signs)

• Different signs? → subtract the numbers → put back sign of

(without the signs) larger (absolute value)

number

136
Objective 4: Simplify algebraic expressions using the four basic operations.

Tip:

1. You can only add and subtract like terms.

2. Note that 𝑥 and 𝑥 2 are unlike terms and therefore, cannot be added.

Question 1:

Simplify 2𝑥 + 3𝑥 + 4𝑦 + 5𝑦.

Solution:

2𝑥 + 3𝑥 + 4𝑦 + 5𝑦 = 5𝑥 + 9𝑦

Question 2:

Simplify 3𝑥 2 + 2𝑥 + 5𝑥 + 10.

Solution:

3𝑥 2 + 2𝑥 + 5𝑥 + 10 = 3𝑥 2 + 7𝑥 + 10

137
Objective 5: Substitute numbers for variables in algebraic expressions.

Example:

1
Given that 𝑎 = 8, 𝑏 = −5 and 𝑐 = 2 , find the value of:

1. 𝑎𝑏𝑐

2. 2𝑎 − 𝑏

Solutions:

1. = 𝑎𝑏𝑐

1 Tip: Write the steps of your


= (8)(−5) (2) working going down the
1 page. One equal sign per line.
= (−40) × 2

= −20

2. = 2𝑎 − 𝑏

= 2(8) − (−5)

= 16 + 5

= 21

138
Objective 6: Evaluate expressions involving binary operations (other than the four basic

operations).

Example:

What is the value of 2 ∗ 3 if 𝑎 ∗ 𝑏 = 2𝑎 − 𝑏?

Solution:

𝑎 ∗ 𝑏 = 2𝑎 − 𝑏

2 ∗ 3 = 2(2) − 3

2∗3= 4−3

2∗3= 1

∴2∗3= 1

Question:

𝑎2 −𝑏
Given 𝑎 ∗ 𝑏 = , find the value of 5 ∗ 9.
2√𝑏

Solution:

𝑎2 −𝑏
𝑎∗𝑏 = 2√𝑏

(5)2 −(9)
5∗9=
2√(9)

25−9
5∗9= 2(3)

16
5∗9= 6

8
5∗9= 3

139
Objective 7: Apply the distributive law to factorise or expand algebraic expression.

Distributive Law

➢ 𝑎(𝑏 + 𝑐) = 𝑎𝑏 + 𝑎𝑐

Factorising an algebraic expression

Example:

Factorize completely 6𝑎2 − 9𝑎.

Solution:

6𝑎2 − 9𝑎 = 3𝑎(2𝑎 − 3)

Expanding an algebraic expression

Example:

Expand 3𝑥(𝑥 − 6𝑦).

Solution:

= 3𝑥(𝑥 − 6𝑦)

= 3𝑥 2 − 18𝑥𝑦

140
Objective 8: Simplify algebraic fractions.

Example:

Express the following as one fraction:

𝑥+1 𝑥
−8
6

Solution:
𝑥+1 𝑥
= −8
6

4(𝑥+1)−3(𝑥)
= 24

4𝑥+4−3𝑥
= 24

𝑥+4
= 24

Flow of your working:

𝐴 𝐵
Consider 𝑥 + 𝑦 .

The LCM of 𝑥 and 𝑦 is 𝑥𝑦.

Therefore,

𝐴 𝐵
=𝑥+𝑦

𝑦𝐴+𝑥𝐵
= 𝑥𝑦

Note: The LCM of 𝑥 and 𝑥 2 is 𝑥 2 .

141
Objective 9: Use the laws of indices to manipulate expressions with integral indices

Laws of indices:

𝑥1 = 𝑥

𝑥0 = 1

𝑥 𝑚 ∙ 𝑥 𝑛 = 𝑥 𝑚+𝑛

𝑥𝑚
= 𝑥 𝑚−𝑛
𝑥𝑛

(𝑥 𝑚 )𝑛 = 𝑥 𝑚𝑛

(𝑥𝑦)𝑚 = 𝑥 𝑚 𝑦 𝑚

𝑥 𝑚 𝑥𝑚
(𝑦 ) = 𝑦 𝑚

1
𝑥 −𝑚 = 𝑥 𝑚

142
Objective 10: Solve linear equations in one unknown

Example:

Find the value of 𝑥 when 4𝑥 − 3 = 21.

Solution:

4𝑥 − 3 = 21

4𝑥 = 21 + 3

4𝑥 = 24

24
𝑥=
4
Change the operation when “going across” the equal sign

𝑥=6

Equal signs fall under each other

Example:
𝑥+7
Find the value of 𝑥 when = −2.
5

Solution:

𝑥+7
= −2
5

𝑥 + 7 = 5 × −2

𝑥 + 7 = −10

𝑥 = −10 − 7

𝑥 = −17

143
Objective 11: Solve simultaneous linear equations, in two unknowns, algebraically.

You can use the elimination method as well as the substitution method.

Example:

Solve the following pair of simultaneous equations:

3𝑥 + 2𝑦 = 12

2𝑥 + 2𝑦 = 10

Solution:

First, ask yourself:

Question: Are any 𝑥, 𝑦 coefficients the same?

Answer: Yes. The 𝑦 coefficients are the same.

Question: What are the signs in front the coefficients?

Answer: If signs are the same, subtract. But if the signs are different, add.

3𝑥 + 2𝑦 = 12 → Equation 1

2𝑥 + 2𝑦 = 10 → Equation 2

Equation 1 − Equation 2 gives:

1𝑥 + 0𝑦 = 2

𝑥=2

144
Substitute 𝑥 = 2 into Equation 1:

3(2) + 2𝑦 = 12

6 + 2𝑦 = 12

2𝑦 = 12 − 6

2𝑦 = 6

6
𝑦=
2

𝑦=3

∴ 𝑥 = 2 and 𝑦 = 3

145
Objective 12: Solve a simple linear inequality in one unknown.

Note:

Inequalities are generally the same as normal equations EXCEPT that they have

inequality signs.

As such, you need to look out for the two situations in which we need to change the

direction of the inequality sign.

1. Swap both sides (entire LHS ↔ RHS)

2. “×” or “÷” by a negative number

Example 1:
𝑥−4
Find the range of values of 𝑥 that satisfy: ≤ −3
6

Solution:

𝑥−4
≤ −3
6

𝑥 − 4 ≤ −3 × 6

𝑥 − 4 ≤ −18

𝑥 ≤ −18 + 4

𝑥 ≤ −14

Example 2:

Find the range of values of 𝑥 that satisfy: 3𝑥 + 5 < 11𝑥 + 37

Solution:

146
3𝑥 + 5 < 11𝑥 + 37

3𝑥 − 11𝑥 < 37 − 5

−8𝑥 < 32
32
𝑥 > −8

𝑥 > −4

147
Objective 13: Change the subject of formulae.

Example:

(a) Make 𝑟 the subject of EACH of the following formulae:

(i) 𝑟 − ℎ = 𝑟ℎ [2]

(ii) 𝑉 = 𝜋𝑟 2 ℎ [2]

Solution:

(i) 𝑟 − ℎ = 𝑟ℎ

𝑟 − 𝑟ℎ = ℎ

𝑟(1 − ℎ) = ℎ


𝑟 = 1−ℎ

(ii) 𝑉 = 𝜋𝑟 2 ℎ
𝑉
= 𝑟2
𝜋ℎ

𝑉
√ =𝑟
𝜋ℎ

148
Objective 14: Factorise algebraic expressions.

We will be focussing on FOUR (4) types of factorization:

1. Basic

2. Difference of two squares

3. Four Terms (Grouping)

4. Quadratic

Basic

Example: 4𝑎 + 2𝑏 = 2(2𝑎 + 𝑏)

Difference of two squares

Rule: 𝒂𝟐 − 𝒃𝟐 = (𝒂 + 𝒃)(𝒂 − 𝒃)

Example:

Factorize completely 4𝑥 2 − 100.

Solution:

4𝑥 2 − 100 = 4(𝑥 2 − 25)

Note: First factorize out 4


from both terms and then
Now √𝑥 2 = 𝑥 and √25 = 5 factorize using the
difference of two squares.

∴ 4𝑥 2 − 100 = 4(𝑥 − 5)(𝑥 + 5)

149
Four Terms (Grouping)

Example:

Factorize the expression 2𝑥 2 + 8𝑥 − 3𝑥 − 12.

Solution:

= 2𝑥 2 + 8𝑥 − 3𝑥 − 12

= 2𝑥(𝑥 + 4) − 3(𝑥 + 4)

= (2𝑥 − 3)(𝑥 + 4)

Quadratic

Example:

Factorize 2𝑥 2 + 5𝑥 + 2.

Solution:

𝑎×𝑐 =2×2

𝑎×𝑐 =4

We need to play the factorizing game and find two values that multiply to give 4 but add

to give 5.

= 2𝑥 2 + 5𝑥 + 4

= 2𝑥 2 + 1𝑥 + 4𝑥 + 4

= 𝑥(2𝑥 + 1) + 2(2𝑥 + 2)

= (𝑥 + 2)(2𝑥 + 1)

150
Objective 15: Rewrite a quadratic expression in the form 𝑎(𝑥 + ℎ)2 + 𝑘.

In algebra, completing the square is a technique used to convert a quadratic equation of

the form 𝑎𝑥 2 + 𝑏𝑥 + 𝑐 where 𝑎, 𝑏 and 𝑐 are constants

into the form 𝒂(𝒙 + 𝒉)𝟐 + 𝒌 where 𝑎, ℎ and 𝑘 are constants.

We will look at two methods used to complete the square.

• Formula Method

• Natural Method - “making a perfect square”

Two other methods we will look at are: Comparing coefficients and Calculator Method

Formula Method:

For the formula method, you are required to memorise two formulas.

They are:

𝒃
1. 𝒉 = 𝟐𝒂

𝟒𝒂𝒄−𝒃𝟐 𝒃𝟐
2. 𝒌 = OR 𝒌 = 𝒄 − 𝟒𝒂
𝟒𝒂

Example:

Express 2𝑥 2 + 4𝑥 − 1 in the form 𝑎(𝑥 + ℎ)2 + 𝑘.

151
Solution:

2𝑥 2 + 4𝑥 − 1 is of the form 𝑎𝑥 2 + 𝑏𝑥 + 𝑐

where 𝑎 = 2, 𝑏 = 4 and 𝑐 = −1

𝑏 4𝑎𝑐−𝑏2
ℎ = 2𝑎 𝑘= 4𝑎

Note: Write out


4 4(2)(−1)−(4)2 your calculations
= 2(2) = 4(2)
for ℎ and 𝑘 side by
side as shown here.
−8−16
=1 = 8

= −3

∴ 2𝑥 2 + 4𝑥 − 1 = 2(𝑥 + 1)2 − 3

Natural Method:

Example:

Express 2𝑥 2 + 4𝑥 − 1 in the form 𝑎(𝑥 + ℎ)2 + 𝑘.

Solution:

= 2𝑥 2 + 4𝑥 − 1

Factor out 𝒂 from the first two terms.

= 2(𝑥 2 + 2𝑥) − 1

152
𝟏
Find 𝟐 of 𝒃 and square it. Add it to form a perfect square and then subtract the added term.

= 2(𝑥 2 + 2𝑥 + 1) − 1 − 2(1)

Simplify until the quadratic is of the form 𝒂(𝒙 + 𝒉)𝟐 + 𝒌

= 2(𝑥 + 1)2 − 3

∴ 2𝑥 2 + 4𝑥 − 1 = 2(𝑥 + 1)2 − 3

Comparing Coefficients Method:

Example:

Express 2𝑥 2 + 4𝑥 − 1 in the form 𝑎(𝑥 + ℎ)2 + 𝑘.

Solution:

2𝑥 2 + 4𝑥 − 1 = 𝑎(𝑥 + ℎ)2 + 𝑘

2𝑥 2 + 4𝑥 − 1 = 𝑎(𝑥 2 + 2𝑥ℎ + ℎ2 ) + 𝑘

2𝑥 2 + 4𝑥 − 1 = 𝑎𝑥 2 + 2𝑎𝑥ℎ + 𝑎ℎ2 + 𝑘

2𝑥 2 + 4𝑥 − 1 = 𝑎𝑥 2 + (2𝑎ℎ)𝑥 + (𝑎ℎ2 + 𝑘)

Comparing coefficients of 𝑥 2 gives:

𝑎=2

153
Comparing coefficients of 𝑥 gives:

2𝑎ℎ = 4

2(2)ℎ = 4

4ℎ = 4

ℎ=1

Comparing constants gives:

𝑎ℎ2 + 𝑘 = −1

(2)(1)2 + 𝑘 = −1

2 + 𝑘 = −1

𝑘 = −1 − 2

𝑘 = −3

∴ 2𝑥 2 + 4𝑥 − 1 = 2(𝑥 + 1)2 − 3

154
Objective 16: Solve quadratic equations algebraically.

Example:

Solve 𝑥 2 + 𝑥 − 42 = 0.

Solution:

𝑥 2 + 𝑥 − 42 = 0

𝑥 2 + 7𝑥 − 6𝑥 − 42 = 0

𝑥(𝑥 + 7) − 6(𝑥 + 7) = 0

(𝑥 − 6)(𝑥 + 7) = 0

Either 𝑥−6=0 or 𝑥+7=0

𝑥=6 𝑥 = −7

155
Objective 17: Solve word problems.

Example:

Four mangoes and two pears cost $24.00, while two mangoes and three pears cost

$16.00.

Let the cost of one mango be represented by 𝑥 and the cost of one pear be represented

by 𝑦.

Using the information provided, this can be represented by the pair of simultaneous

equations:

4𝑥 + 2𝑦 = 24

2𝑥 + 3𝑦 = 16

You can then solve the pair of simultaneous equations to find the cost of 1 mango and 1

pear.

156
Objective 18: Solve a pair of equations in two variables when one equation is quadratic

or non-linear and the other linear.

Example:

𝑦 = 𝑥2 + 𝑥 − 4 → Equation 1

𝑦 = 2𝑥 + 2 → Equation 2

Equating both equations gives:

𝑥 2 + 𝑥 − 4 = 2𝑥 + 2

𝑥 2 + 𝑥 − 2𝑥 − 4 − 2 = 0

𝑥2 − 𝑥 − 6 = 0

𝑥 2 − 3𝑥 + 2𝑥 − 6 = 0

𝑥(𝑥 − 3) + 2(𝑥 − 3) = 0

(𝑥 + 2)(𝑥 − 3) = 0

Either 𝑥+2=0 or 𝑥−3=0

𝑥 = −2 𝑥=3

When 𝑥 = −2,

𝑦 = 2(−2) + 2

𝑦 = −4 + 2

𝑦 = −2

157
When 𝑥 = 3,

𝑦 = 2(3) + 2

𝑦 = 6+2

𝑦=8

∴ 𝑥 = −2, 𝑦 = −2 and 𝑥 = 3, 𝑦 = 8

158
Objective 19: Prove two algebraic expressions to be identical.

You should be familiar with the following algebraic expressions:

1. (𝑎 + 𝑏)2 = 𝑎2 + 2𝑎𝑏 + 𝑏 2

2. (𝑎 − 𝑏)2 = 𝑎2 − 2𝑎𝑏 + 𝑏 2

3. (𝑎 + 𝑏)(𝑎 − 𝑏) = 𝑎2 − 𝑏 2

159
Objective 20: Represent direct and inverse variation symbolically.

Objective 21: Solve problems involving direct variation and inverse variation.

Directly Proportionality

Note:
• The graph denotes two variables
which are said to be proportional or
directly proportional to each other.
• As 𝑥 increases, 𝑦 also increases by
a factor of 𝑥. In this case, the factor
is 1.

𝑥
𝑃𝑟𝑜𝑝𝑜𝑟𝑡𝑖𝑜𝑛𝑎𝑙

If 𝑦 is directly proportional to 𝑥, this is written as

𝑦∝𝑥

Expressing this as an equation, we get

𝒚 = 𝒌𝒙

160
Inverse Proportionality

Note:
• The graph denotes two
variables which are said to be
inversely proportional to each
other.
1 • As 𝑥 increases, 𝑦 decreases.
𝑦=𝑥

𝑥
𝐼𝑛𝑣𝑒𝑟𝑠𝑒𝑙𝑦 𝑃𝑟𝑜𝑝𝑜𝑟𝑡𝑖𝑜𝑛𝑎𝑙

If 𝑦 is indirectly proportional to 𝑥, this is written as


1
𝑦∝𝑥

Expressing this as an equation, we get

𝒌
𝒚=𝒙

161
Question 1:

Consider the table below:

𝑥 3 4 ______

𝑦 12 ______ 36

Find the missing values given that 𝑦 is directly proportional to 𝑥.

Solution:

𝑦∝𝑥

𝑦 = 𝑘𝑥

(12) = 𝑘(3)

12
=𝑘
3

4=𝑘

So, we have 𝑦 = 4𝑥.

When 𝑥 = 4,

𝑦 = 4(4)

𝑦 = 16

When 𝑦 = 36,

36 = 4𝑥

36
=𝑥
4

9=𝑥

162
Question 2:

When 𝑥 = 15, 𝑦 = 3.

Find the value of 𝑦 when 𝑥 = 30, given that 𝑦 is directly proportional to 𝑥.

Solution:

𝑦∝𝑥

𝑦 = 𝑘𝑥

(3) = 𝑘(15)

3
=𝑘
15

1
=𝑘
5

1
So, we have 𝑦 = 5 𝑥.

When 𝑥 = 30,
1
𝑦 = 5 (30)

𝑦=6

163
Past Paper Question – January 2009 – Paper 2 – Question 2

(a) Simplify, expressing your answer as a single fraction

2𝑚 5𝑚
− [3]
𝑛 3𝑛

(b) If 𝑎 ∗ 𝑏 = 𝑎2 − 𝑏, evaluate 5 ∗ 2. [1]

(c) Factorize completely 3𝑥 − 6𝑦 + 𝑥 2 − 2𝑥𝑦. [2]

(d) A drinking straw of length 21 𝑐𝑚 is cut into 3 pieces.

The length of the first piece is 𝑥 𝑐𝑚.

The second piece is 3 𝑐𝑚 shorter than the first piece.

The third piece is twice as long as the first piece.

(i) State, in terms of 𝑥, the length of EACH of the pieces. [2]

(ii) Write an expression, in terms of 𝑥, to represent the sum of the lengths of the

three pieces of drinking straw. [1]

(iii) Hence, calculate the value of 𝑥. [3]

Total: 12 marks

164
Past Paper Question – January 2009 – Paper 2 – Question 2 – Solution

(a) Simplify, expressing your answer as a single fraction

2𝑚 5𝑚
− [3]
𝑛 3𝑛

2𝑚 5𝑚 6𝑚−5𝑚
− =
𝑛 3𝑛 3𝑛

2𝑚 5𝑚 𝑚
− = 3𝑛
𝑛 3𝑛

(b) If 𝑎 ∗ 𝑏 = 𝑎2 − 𝑏, evaluate 5 ∗ 2. [1]

𝑎 ∗ 𝑏 = 𝑎2 − 𝑏

5 ∗ 2 = (5)2 − (2)

5 ∗ 2 = 25 − 2

5 ∗ 2 = 23

(c) Factorize completely 3𝑥 − 6𝑦 + 𝑥 2 − 2𝑥𝑦. [2]

= 3𝑥 − 6𝑦 + 𝑥 2 − 2𝑥𝑦

= 3(𝑥 − 2𝑦) + 𝑥(𝑥 − 2𝑦)

= (3 + 𝑥)(𝑥 − 2𝑦)

(d) A drinking straw of length 21 𝑐𝑚 is cut into 3 pieces.

The length of the first piece is 𝑥 𝑐𝑚.

The second piece is 3 𝑐𝑚 shorter than the first piece.

The third piece is twice as long as the first piece.

165
(i) State, in terms of 𝑥, the length of EACH of the pieces. [2]

First piece = 𝑥

Second piece = 𝑥 − 3

Third piece = 2𝑥

(ii) Write an expression, in terms of 𝑥, to represent the sum of the lengths of the

three pieces of drinking straw. [1]

We have,

𝑥 + 𝑥 − 3 + 2𝑥 = 21

4𝑥 − 3 = 21

(iii) Hence, calculate the value of 𝑥. [3]

4𝑥 − 3 = 21

4𝑥 = 21 + 3

4𝑥 = 24

24
𝑥= 4

𝑥=6

Total: 12 marks

166
Past Paper Question – June 2023 – Paper 2 – Question 2

4 15𝑥
2. (a) Simplify 5𝑥 × . [1]
16

(b) Solve the inequality 12 − 4𝑚 ≤ 5 − 8𝑚. [2]

(c) The diagram below shows a compound shape, 𝐿𝑀𝑁𝑃𝑄𝑅, made from two

rectangles. The lengths in the diagram, which are written in terms of 𝑥, are in

centimetres.

𝑳 𝟒𝒙 − 𝟓 𝑴

𝒙+𝟑

𝟑𝒙
𝑸
𝑹

𝑷 𝑵
𝒙+𝟏

(i) Find an expression, in terms of 𝑥, for the length

(a) 𝑃𝑄 [1]

(b) 𝑅𝑄 [1]

(ii) Given that the TOTAL area of the shape is 414 𝑐𝑚2 , show that

𝑥 2 + 𝑥 − 72 = 0. [4]

Total: 9 marks

167
Past Paper Question – June 2023 – Paper 2 – Question 2 – Solution

4 15𝑥
2. (a) Simplify 5𝑥 × . [1]
16

3
4 15𝑥 3
× =4
5𝑥 16
4

(b) Solve the inequality 12 − 4𝑚 ≤ 5 − 8𝑚. [2]

12 − 4𝑚 ≤ 5 − 8𝑚

−4𝑚 + 8𝑚 ≤ 5 − 12

4𝑚 ≤ −7
7
𝑚 ≤ −4

(c) The diagram below shows a compound shape, 𝐿𝑀𝑁𝑃𝑄𝑅, made from two

rectangles. The lengths in the diagram, which are written in terms of 𝑥, are in

centimetres.

𝑳 𝟒𝒙 − 𝟓 𝑴

𝒙+𝟑 𝑨

𝟑𝒙
𝑸
𝑹

𝑷 𝑵
𝒙+𝟏

168
(i) Find an expression, in terms of 𝑥, for the length

(a) 𝑃𝑄 [1]

𝑃𝑄 = 𝑀𝑁 − 𝐿𝑅

𝑃𝑄 = 3𝑥 − (𝑥 + 3)

𝑃𝑄 = 3𝑥 − 𝑥 − 3

𝑃𝑄 = 2𝑥 − 3

(b) 𝑅𝑄 [1]

𝑅𝑄 = 𝐿𝑀 − 𝑃𝑁

𝑅𝑄 = 4𝑥 − 5 − (𝑥 + 1)

𝑅𝑄 = 4𝑥 − 5 − 𝑥 − 1

𝑅𝑄 = 3𝑥 − 6

(ii) Given that the TOTAL area of the shape is 414 𝑐𝑚2 , show that

𝑥 2 + 𝑥 − 72 = 0. [4]

Area of section 𝐴 = 𝐿𝑀 × 𝐿𝑅

Area of section 𝐴 = (4𝑥 − 5)(𝑥 + 3)

Area of section 𝐴 = 4𝑥 2 + 12𝑥 − 5𝑥 − 15

Area of section 𝐴 = 4𝑥 2 + 7𝑥 − 15

Area of section 𝐵 = 𝑃𝑄 × 𝑃𝑁

Area of section 𝐵 = (2𝑥 − 3)(𝑥 + 1)

Area of section 𝐵 = 2𝑥 2 + 2𝑥 − 3𝑥 − 3

Area of section 𝐵 = 2𝑥 2 − 𝑥 − 3
169
So, we have,

Total area of the shape = Area of section 𝐴 + Area of section 𝐵

Total area of the shape = 4𝑥 2 + 7𝑥 − 15 + 2𝑥 2 − 𝑥 − 3

Total area of the shape = 6𝑥 2 + 6𝑥 − 18

Since the total area of the shape is 414 𝑐𝑚2, we have,

6𝑥 2 + 6𝑥 − 18 = 414

6𝑥 2 + 6𝑥 − 18 − 414 = 0

6𝑥 2 + 6𝑥 − 432 = 0

(÷ 6)

𝑥 2 + 𝑥 − 72 = 0
Q.E.D.

Total: 9 marks

170
Section 7 – Relations, Functions and Graphs

Objective 1: Explain basic concepts associated with relations.

To establish/define a relation we need the following:

1. Two sets of data

2. Rule between them

3. Direction of “relation”/rule

An arrow diagram is drawn below:

Rule: 𝑦 = 6𝑥
𝒙 𝒚

6
V

1
NOTE:
2 12
V

The two sets of data are 𝒙 and 𝒚.


3 18
V

The rule between them is 𝒚 = 6𝒙.


4 24
V

The direction of the relation is going


5 30 from the domain to the co-domain.
V

input output
also called also called
domain co-domain / range

171
Types of Relations

There are FOUR types:

One to One

𝒙 𝒚
NOTE:
∎ ∎ One to One:-
V

One input is mapped


∎ ∎ unto 1 output.
V

In other words, one


∎ ∎ output came from one
V

input.

input output

𝒙 One to Many 𝒚

∎ ∎
V

NOTE:
One to Many:-
∎ ∎
V

One input is mapped


unto 2 or more output.

input output

Many to One
𝒙 𝒚

∎ ∎
V

NOTE:
Many to One:-
∎ ∎
V

Two or more input is


mapped unto 1 output.

input output

𝒙 𝒚
172
Many to Many

NOTE:
∎ ∎ Many to Many:-

V
Two or more input is mapped
∎ ∎ unto 1 output

V
AND
∎ ∎ One input is mapped unto 2 or
V more output.

input output

173
Objective 2: Represent a relation in various ways.

An arrow diagram is drawn below.

Rule: 𝒚 = 𝟒𝒙 − 𝟔
𝒙 𝒚
V
4 10
V

5 14
V

6 18
V

7 22

input output

The set of ordered pairs would be {(4, 10), (5, 14), (6, 18), (7, 22)}.

174
Objective 3: State the characteristics that define a function.

Functions:

• Functions are based upon the concept of substitution.

• Relations that are functions:

1. One to One

2. Many to One NOTE:


Not every relation is
• Relations that are NOT functions: a function BUT every
function is a relation.
1. Many to Many

2. One to Many

Functions are based on the concept of inputting values into a function and getting an

output.

𝟑 𝒇(𝒙) = 𝒙 + 𝟓 𝟖

input output
function

𝑓(𝑥) is read as “𝑓 𝑜𝑓 𝑥". It is a function where 𝑥 is the input.

In the Figure above, if 𝟑 is inputted into the function, the output will be 𝟖.

175
Objective 4: Use functional notation.

Notation Explanation

𝑓: 𝑥 → 𝑥 2 or 𝑓(𝑥) = 𝑥 2 The element 𝑥 in the domain is mapped onto the

element 𝑥 2 in the codomain.

𝑓 −1 (𝑥) This is the inverse of 𝑓(𝑥).

𝑓𝑔 = 𝑓[𝑔(𝑥)] This is a composite function where you substitute the

function 𝑔 into the function 𝑓.

Question 1:
𝑥+10
𝑓(𝑥) = 2𝑥 − 9 𝑔(𝑥) = ,𝑥 ≠ 0
𝑥

Find:

(a) 𝑓(2)

(b) 𝑔(−4)

(c) 𝑓 −1 (𝑥)

(d) 𝑔−1 (𝑥)

(e) 𝑔𝑓(𝑥)

(f) 𝑔𝑓(3)

Solution:

(a) 𝑓(𝑥) = 2𝑥 − 9

𝑓(2) = 2(2) − 9

𝑓(2) = 4 − 9

𝑓(2) = −5

176
𝑥+10
(b) 𝑔(𝑥) = 𝑥

(−4)+10
𝑔(−4) = (−4)

−6
𝑔(−4) = −4

3
𝑔(−4) = 2

(c) 𝑓(𝑥) = 2𝑥 − 9

Let 𝑦 = 𝑓(𝑥).

𝑦 = 2𝑥 − 9

Interchanging variables.

𝑥 = 2𝑦 − 9

Make 𝑦 the subject of the formula.

𝑥 + 9 = 2𝑦

𝑥+9
=𝑦
2

𝑥+9
∴ 𝑓 −1 (𝑥) = 2

𝑥+10
(d) 𝑔(𝑥) = 𝑥

Let 𝑦 = 𝑔(𝑥).
𝑥+10
𝑦= 𝑥

177
Interchanging variables.
𝑦+10
𝑥= 𝑦

Make 𝑦 the subject of the formula.

𝑥𝑦 = 𝑦 + 10

𝑥𝑦 − 𝑦 = 10

𝑦(𝑥 − 1) = 10
10
𝑦 = 𝑥−1

10
∴ 𝑔−1 (𝑥) = 𝑥−1

(e) 𝑔𝑓(𝑥) = 𝑔[𝑓(𝑥)]

𝑔𝑓(𝑥) = 𝑔(2𝑥 − 9)

(2𝑥−9)+10
𝑔𝑓(𝑥) = (2𝑥−9)

2𝑥+1
𝑔𝑓(𝑥) = 2𝑥−9

2𝑥+1
(f) 𝑔𝑓(𝑥) = 2𝑥−9

2(3)+1
𝑔𝑓(3) = 2(3)−9

6+1
𝑔𝑓(3) = 6−9

7
𝑔𝑓(3) = −3

178
Question 2:
𝑥+1
ℎ(𝑥) = 𝑥+3

Find the value of 𝑥 for which the function is undefined.

Solution:

A function is undefined when the denominator is equal to zero.

𝑥+3=0

𝑥 = −3

Question 3:
4
𝑓(𝑥) = 2𝑥+1

Find the value of 𝑥 for which the function is undefined.

Solution:

A function is undefined when the denominator is equal to zero.

2𝑥 + 1 = 0

2𝑥 = −1
1
𝑥 = −2

179
Objective 5: Distinguish between a relation and a function.

Vertical line test

The vertical line test is used to determine whether a mapping is a function.

If you can draw a vertical line through any part of the graph of the function and it cuts

the graph at most once, then it is a function. If it cuts the graph more than once, it is not

a function.

Example: 𝑦 = 𝑥 + 3 Example: 𝑦 = 𝑥 2 + 1

180
Multiple Choice Question

Which of the following does represents the graph of a function?

𝒚 𝒚
(A) (B)

𝒙 𝒙

(C) 𝒚 (D) 𝒚

𝒙 𝒙

Answer: B

181
Objective 6: Draw graphs of linear functions.

There are 3 main types of lines that you must be able to draw.

1. 𝑦 = 𝑎𝑥 where 𝑎 is a constant

2. 𝑎𝑥 + 𝑏𝑦 = 𝑐 where 𝑎, 𝑏, 𝑐 are constants

3. 𝑥 = 𝑎 or 𝑦 = 𝑎 where 𝑎 is a constant

1. Consider 𝒚 = 𝒙.

Some points on this line would be (1, 1) , (2, 2) , (3, 3) , etc.

182
Example:

𝟏
Consider 𝒚 = 𝟐 𝒙.

1 3
Some points on this line would be (1, 2) , (2, 1) , (3, 2) , etc.

Consider 𝒚 = 𝟐𝒙.

Some points on this line would be (1,2) , (2, 4) , (3,6) , etc.

183
2. Consider 𝒙 + 𝒚 = 𝟓.

When 𝑥 = 0, 𝑦 = 5.

When 𝑦 = 0, 𝑥 = 5.

So two points on the graph are (0, 5) and (5, 0).

184
3. Consider 𝒙 = 𝟓.

This is a vertical line that cuts the 𝑥-axis at (5, 0).

Along this line, the 𝑥-coordinate will always be 5.

Some points on this line are (5, 0) , (5, 1) , (5, 2) , etc.

𝒙=𝟓

185
Consider 𝒚 = 𝟑.

This is a horizontal line that cuts the 𝑦-axis at (0, 3).

Along this line, the 𝑦-coordinate will always be 3.

Some points on this line are (0, 3) , (1, 3) , (2, 3) , etc.

𝒚=𝟑

186
Objective 7: Determine the intercepts of the graph of linear functions.

To determine the 𝒙-intercepts of the graph of a linear function, let 𝒚 = 𝟎.

To determine the 𝒚-intercepts of the graph of a linear function, let 𝒙 = 𝟎.

Example:

Determine the 𝑥 and 𝑦 intercepts of the line 2𝑥 + 3𝑦 = 18.

Solution:

When 𝑦 = 0,

2𝑥 + 3(0) = 18

2𝑥 = 18

18
𝑥= 2

𝑥=9

∴ The 𝑥-intercept is (9, 0).

When 𝑥 = 0,

2(0) + 3𝑦 = 18

3𝑦 = 18
18
𝑦= 3

𝑦=6

∴ The 𝑦-intercept is (0, 6).

187
Objective 8: Determine the gradient of a straight line.

➢ Another word for gradient is “slope” or “steepness”.

➢ The gradient of a straight line tells us how steep a line is, therefore, the bigger the

gradient, the steeper the line.

Formula:

The gradient can be calculated as the change in 𝑦 over the change in 𝑥.

𝑅𝑖𝑠𝑒
Gradient = 𝑅𝑢𝑛

𝛥𝑦
Gradient = 𝛥𝑥

𝒚𝟐 −𝒚𝟏
Gradient, 𝒎 =
𝒙𝟐 −𝒙𝟏

Gradient Representation of Line

Positive

Negative

Zero

Infinite
(undefined)

188
Objective 9: Determine the equation of a straight line.

The equation of a straight line is of the form:

𝒚 = 𝒎𝒙 + 𝒄

where

𝑚 = gradient of the line

𝑐 = y-intercept

If given the gradient 𝑚 and a point (𝑥1 , 𝑦1 ), use the following formula to find the equation

of the line:

𝒚 − 𝒚𝟏 = 𝒎(𝒙 − 𝒙𝟏 )

189
Objective 10: Solve problems involving the gradient of parallel and perpendicular lines.

Parallel Lines

Parallel lines are a fixed distance apart and will never meet, no matter how long they are

extended.

𝑚1

𝑚2

Lines that are parallel have the same gradient. Therefore, for parallel lines 𝒎𝟏 = 𝒎𝟐 .

Perpendicular Lines

Two lines are perpendicular if one is at right angles to each other.

𝑚2

𝑚1

If two lines are perpendicular, then their gradients will multiply together to give -1.

𝟏
Therefore, for perpendicular lines, 𝒎𝟐 = − 𝒎 .
𝟏

190
Objective 11: Determine from co-ordinates on a line segment:

(a) the length

(b) the co-ordinates of the midpoint

To calculate the distance between two points in a straight line, we use the distance

formula below which is an application of Pythagoras’ Theorem.

Distance Formula:

Distance = √(𝒙𝟐 − 𝒙𝟏 )𝟐 + (𝒚𝟐 − 𝒚𝟏 )𝟐

To calculate the midpoint, the following formula is used:

Midpoint Formula:

𝒙𝟏 +𝒙𝟐 𝒚𝟏 +𝒚𝟐
Midpoint = ( , )
𝟐 𝟐

191
Question:

Consider the graph below:

𝑦
𝐴(3, 8)
×

𝐶(−2, 2)
× 𝐵(9, 1)
×
𝑥

(a) Determine the equation of line 𝐴𝐵. [2]

(b) Find the midpoint of 𝐴𝐵. [1]

(c) Find the equation of the line perpendicular to 𝐴𝐵 passing through 𝐶. [3]

Solution:

(a) Points are 𝐴(3, 8) and 𝐵(9, 1).

𝑦 −𝑦
Gradient = 𝑥2−𝑥1
2 1

1−8
Gradient = 9−3

7
Gradient = − 6

7
Substituting 𝑚 = − 6 and point (3, 8) into 𝑦 − 𝑦1 = 𝑚(𝑥 − 𝑥1 ) gives:

192
7
𝑦 − 8 = − 6 (𝑥 − 3)

7 7
𝑦 − 8 = −6𝑥 + 2

7 7
𝑦 = −6𝑥 + 2 + 8

7 23
𝑦 = −6𝑥 + 2

7 23
∴ The equation of line 𝐴𝐵 is: 𝑦 = − 6 𝑥 + 2

(b) Points are 𝐴(3, 8) and 𝐵(9, 1).

𝑥1 +𝑥2 𝑦1 +𝑦2
Midpoint = ( , )
2 2

3+9 8+1
Midpoint = ( , )
2 2

12 9
Midpoint = ( 2 , 2)

9
Midpoint = (6 , 2)

7
(c) Gradient of line 𝐴𝐵 = − 6

6
Gradient of perpendicular line = 7

6
Substituting 𝑚 = 7 and point (−2, 2) into 𝑦 − 𝑦1 = 𝑚(𝑥 − 𝑥1 ) gives:

6
𝑦 − 2 = 7 (𝑥 − (−2))

6 12
𝑦 − 2 = 7𝑥 + 7

6 12
𝑦 = 7𝑥 + +2
7

6 26
𝑦 = 7𝑥 + 7

193
Objective 12: Solve a pair of simultaneous linear equations in two unknowns graphically.

Example:

𝑦 = 4 − 5𝑥 → Equation 1

𝑦 = 3𝑥 − 4 → Equation 2

Solution:

𝑦 = 4 − 5𝑥 𝑦 = 3𝑥 − 4

The point of intersection is (1, −1).

194
Objective 13: Represent the solution of linear inequalities in one variable using:

(a) set notation

(b) the number line

(c) graph

Set Notation

The solution set is {𝑥: −1 < 𝑥 < 1}.

The Number Line

Consider the inequality −4 ≤ 𝑥 < 3.

Graph

Consider the inequality 𝑥 < 4.

195
Objective 14: Draw a graph to represent a linear inequality in two variables.

Objective 15: Use linear programming techniques to graphically solve problems involving

two variables.

In linear programming, you will have to identify the region that satisfy the given

inequalities.

Identify the region that satisfies both of the following inequalities:

𝑥+𝑦 ≤5

𝑦≤𝑥

196
Objective 16: Derive the composition of functions.

A composite function is created when one function is substituted into another function.

Example:

𝑓(𝑥) = 3𝑥 + 1

𝑔 (𝑥 ) = 𝑥 2

Determine:

(a) 𝑓𝑔(𝑥)

(b) 𝑔𝑓(𝑥)

Solution:

(a) 𝑓𝑔(𝑥) = 3𝑥 2 + 1 [Replace 𝑥 with 𝑥 2 ]

(b) 𝑔𝑓(𝑥) = (3𝑥 + 1)2 [Replace 𝑥 with (3𝑥 + 1)]

𝑔𝑓(𝑥) = 9𝑥 2 + 6𝑥 + 1

197
Objective 17: State the relationship between a function and its inverse.

The relationship between a function and its inverse is:

𝒇𝒇−𝟏 (𝒙) = 𝒙

𝒇−𝟏 𝒇(𝒙) = 𝒙

Example:

𝑓(𝑥) = 2𝑥 + 9

𝑥−9
𝑓 −1 (𝑥) =
2

Find 𝑓𝑓 −1 (𝑥) and 𝑓 −1 𝑓(𝑥).

Solution:

𝑥−9
𝑓𝑓 −1 (𝑥) = 2 ( )+9
2

𝑓𝑓 −1 (𝑥) = 𝑥 − 9 + 9

𝑓𝑓 −1 (𝑥) = 𝑥

Similarly,

(2𝑥−9)+9
𝑓 −1 𝑓(𝑥) = 2

2𝑥
𝑓 −1 𝑓(𝑥) = 2

𝑓 −1 𝑓(𝑥) = 𝑥

198
Objective 18: Derive the inverse of a function.

There are three steps involved in finding the inverse of a function:

1. Let 𝑓(𝑥) = 𝑦.

2. Interchange the variables.

3. Make 𝑦 the subject of the formula.

STEP 3:
Make 𝒚 the subject
of the formula.

STEP 2:
Interchange
the variables.

STEP 1:
Let 𝒇(𝒙) = 𝒚.

199
Objective 19: Evaluate a function 𝑓(𝑥) at a given value of 𝑥.

Example:

The function 𝑓 is defined as

𝑓: 𝑥 → 3 − 2𝑥.

The diagram below shows the mapping diagram of the function, 𝑓. Determine the value

of 𝑎.

𝟑 − 𝟐𝒙
𝒙 𝒇(𝒙)

-1 𝒂

0 3

1 1

2 -1

Solution:

𝑓(𝑥) = 3 − 2𝑥

𝑓(−1) = 3 − 2(−1)

𝑎 = 3+2

𝑎=5

200
Objective 20: Draw and use the graph of a quadratic function to identify its features:

(a) an element of the domain that has a given image

(b) the image of a given element in the domain

(c) the maximum or minimum value of the function

(d) the equation of the axis of symmetry

Objective 21: Interpret the graph of a quadratic function to determine:

(a) the interval of the domain for which the elements of the range may be

greater than or less than a given point

(b) an estimate of the value of the gradient at a given point

(c) intercepts of the function

Objective 22: Determine the equation of the axis of symmetry and the maximum or

minimum value of a quadratic function expressed in the form 𝑎(𝑥 + ℎ)2 +

𝑘.

Points to note

• When 𝑎 is negative, the graph has a maximum point.

• When 𝑎 is positive, the graph has a minimum point.

The equation of the line of symmetry is: 𝒙 = −𝒉

201
Objective 23: Sketch the graph of a quadratic function expressed in the form 𝑦 =

𝑎(𝑥 + ℎ)2 + 𝑘and determine the number of roots.

You may be asked to find the sketch the graph of the quadratic equation.

Helpful Tips:

➢ Just put the key points.

➢ Draw the curve first, whether it is minimum or maximum.

➢ Draw the 𝑥-axis and then draw the 𝑦-axis accordingly.

Sketch of 𝒙𝟐 + 𝟐𝒙 − 𝟑:

𝑥
−3 1

−3

×
(−1, −4)

202
Objective 24: Draw graphs of non-linear functions.

Objective 25: Interpret graphs of functions.

Objective 26: Solve problems involving graphs of linear and non-linear functions.

Tips:

1. When drawing graphs, ensure to use at least 80% of the graph page.

2. Ensure to draw a smooth curve for non-linear functions.

203
Past Paper Question – January 2009 – Paper 2 – Question 9

(a) Make 𝑡 the subject of the formula

𝑝 𝑡+𝑟
=√ [3]
2 𝑔

(b) (i) Express the function 𝑓(𝑥) = 2𝑥 2 − 4𝑥 − 13 in the form

𝑓(𝑥) = 𝑎(𝑥 + ℎ)2 + 𝑘. [3]

Hence, or otherwise, determine

(ii) the values of 𝑥 at which the graph cuts the 𝑥-axis [4]

(iii) the interval for which 𝑓(𝑥) ≤ 0 [2]

(iv) the minimum value of 𝑓(𝑥) [1]

(v) the value of 𝑥 at which 𝑓(𝑥) is a minimum [2]

Total: 15 marks

204
Past Paper Question – January 2009 – Paper 2 – Question 9 – Solution

(a) Make 𝑡 the subject of the formula

𝑝 𝑡+𝑟
=√ [3]
2 𝑔

𝑝 𝑡+𝑟
=√
2 𝑔

𝑝 2 𝑡+𝑟
( ) =
2 𝑔

𝑝2 𝑡+𝑟
=
4 𝑔

4(𝑡 + 𝑟) = 𝑔𝑝2

4𝑡 + 4𝑟 = 𝑔𝑝2

4𝑡 = 𝑔𝑝2 − 4𝑟

𝑔𝑝2 −4𝑟
𝑡= 4

(b) (i) Express the function 𝑓(𝑥) = 2𝑥 2 − 4𝑥 − 13 in the form

𝑓(𝑥) = 𝑎(𝑥 + ℎ)2 + 𝑘. [3]

2𝑥 2 − 4𝑥 − 13 is in the form 𝑎𝑥 2 + 𝑏𝑥 + 𝑐,

where 𝑎 = 2, 𝑏 = −4 and 𝑐 = −13.

𝑏 4𝑎𝑐−𝑏2
ℎ = 2𝑎 𝑘= 4𝑎

(−4) 4(2)(−13)−(−4)2
ℎ= 𝑘=
2(2) 4(2)

−4 −104−16
ℎ= 𝑘=
4 8

ℎ = −1 𝑘 = −15

205
∴ 𝑓(𝑥) = 2(𝑥 − 1)2 − 15 which is in the form 𝑎(𝑥 + ℎ)2 + 𝑘,

where 𝑎 = 2, ℎ = −1 and 𝑘 = −15.

Hence, or otherwise, determine

(ii) the values of 𝑥 at which the graph cuts the 𝑥-axis [4]

When 𝑦 = 0,

2(𝑥 − 1)2 − 15 = 0

2(𝑥 − 1)2 = 15
15
(𝑥 − 1)2 =
2

15
𝑥 − 1 = ±√ 2

15
𝑥 = 1±√2

∴ The values of 𝑥 at which the graph cuts the 𝑥-axis are

15 15
𝑥 = 1 − √ 2 and 𝑥 = 1 + √ 2 .

(iii) the interval for which 𝑓(𝑥) ≤ 0 [2]

15 15
The interval for which 𝑓(𝑥) ≤ 0 is {𝑥: 1 − √ 2 ≤ 𝑥 ≤ 1 + √ 2 }

(iv) the minimum value of 𝑓(𝑥) [1]

The minimum value of 𝑓(𝑥) is 𝑘 = −15.

206
(v) the value of 𝑥 at which 𝑓(𝑥) is a minimum [2]

The value of 𝑥 at which 𝑓(𝑥) is a minimum is:

𝑥 = −ℎ

𝑥 = −(−1)

𝑥=1

Total: 15 marks

207
Past Paper Question – January 2009 – Paper 2 – Question 10

(a) Two functions are defined as follows:

𝑓: 𝑥 → 𝑥 − 3

𝑔: 𝑥 → 𝑥 2 − 1

(i) Calculate 𝑓(6). [1]

(ii) Find 𝑓 −1 (𝑥). [1]

(iii) Show that 𝑓𝑔(2) = 𝑓𝑔(−2) = 0. [3]

(b) An answer sheet is provided for this question.

The distance-time graph below describes the journey of a train between two train

stations, 𝐴 and 𝐵. Answer the questions below on the answer sheet.

(i) For how many minutes was the train at rest at 𝐵? [1]

208
(ii) Determine the average speed of the train, in 𝑘𝑚/ℎ, on its journey from

𝐴 to 𝐵. [3]

The train continued its journey away from stations 𝐴 and 𝐵 to another station 𝐶,

which is 50 𝑘𝑚 from 𝐵. The average speed on this journey was 60 𝑘𝑚/ℎ.

(iii) Calculate the time, in minutes, taken for the train to travel from 𝐵 to 𝐶. [3]

(iv) On your answer sheet, draw the line segment which describes the journey of

the train from 𝐵 to 𝐶. [3]

Total: 15 marks

209
Past Paper Question – January 2009 – Paper 2 – Question 10 – Solution

(a) Two functions are defined as follows:

𝑓: 𝑥 → 𝑥 − 3

𝑔: 𝑥 → 𝑥 2 − 1

(i) Calculate 𝑓(6). [1]

𝑓(𝑥) = 𝑥 − 3

𝑓(6) = (6) − 3

𝑓(6) = 3

(ii) Find 𝑓 −1 (𝑥). [1]

𝑓 −1 (𝑥) = 𝑥 + 3

(iii) Show that 𝑓𝑔(2) = 𝑓𝑔(−2) = 0. [3]

𝑔(𝑥) = 𝑥 2 − 1

𝑔(2) = (2)2 − 1

𝑔(2) = 4 − 1

𝑔(2) = 3

𝑓𝑔(2) = 𝑓[𝑔(2)]

𝑓𝑔(2) = 𝑓(3)

𝑓𝑔(2) = 3 − 3

𝑓𝑔(2) = 0

210
Now,

𝑔(𝑥) = 𝑥 2 − 1

𝑔(−2) = (−2)2 − 1

𝑔(−2) = 4 − 1

𝑔(−2) = 3

𝑓𝑔(−2) = 𝑓[𝑔(−2)]

𝑓𝑔(−2) = 𝑓(3)

𝑓𝑔(−2) = 3 − 3

𝑓𝑔(−2) = 0

∴ 𝑓𝑔(2) = 𝑓𝑔(−2) = 0
Q.E.D.

211
(b) An answer sheet is provided for this question.

The distance-time graph below describes the journey of a train between two train

stations, 𝐴 and 𝐵. Answer the questions below on the answer sheet.

(i) For how many minutes was the train at rest at 𝐵? [1]

The train was at rest at 𝐵 for 60 − 40 = 20 minutes.

(ii) Determine the average speed of the train, in 𝑘𝑚/ℎ, on its journey from

𝐴 to 𝐵. [3]

Time = 40 minutes
2
Time = 3 hour

212
Distance = 100 𝑘𝑚

Now,

𝐷𝑖𝑠𝑡𝑎𝑛𝑐𝑒
Average speed = 𝑇𝑖𝑚𝑒

100
Average speed = 2
( )
3

Average speed = 150 𝑘𝑚/ℎ

The train continued its journey away from stations 𝐴 and 𝐵 to another station 𝐶,

which is 50 𝑘𝑚 from 𝐵. The average speed on this journey was 60 𝑘𝑚/ℎ.

(iii) Calculate the time, in minutes, taken for the train to travel from 𝐵 to 𝐶. [3]

𝐷𝑖𝑠𝑡𝑎𝑛𝑐𝑒
Time = 𝑆𝑝𝑒𝑒𝑑

50
Time = 60

5
Time = 6 hour

5
Time = 6 × 60 minutes

Time = 50 minutes

(iv) On your answer sheet, draw the line segment which describes the journey of

the train from 𝐵 to 𝐶. [3]

(See graph)

Total: 15 marks

213
Past Paper Question – June 2023 – Paper 2 – Question 8

8. (a) Complete the table for the function 𝑦 = −𝑥 2 + 𝑥 + 7. [2]

𝒙 −3 −2 −1 0 1 2 3 4

1 7 5 −5
𝒚

214
(b) On the grid below, draw the graph of 𝑦 = −𝑥 2 + 𝑥 + 7 for −3 ≤ 𝑥 ≤ 4. [3]

215
(c) Write down the coordinates of the maximum/minimum point of the

graph. [1]

(…………… , ……………)

(d) Write down the equation of the axis of symmetry of the graph. [1]

(e) Use your graph to find the solutions of the equation −𝑥 2 + 𝑥 + 7 = 0. [2]

𝑥 = ………………………………………….. or 𝑥 = …………………………………………..

(f) (i) On the grid on page 24, draw a line through the points (−3, −1) and

(0, 8). [1]

(ii) Determine the equation of this line in the form 𝑦 = 𝑚𝑥 + 𝑐. [2]

Total: 12 marks

216
Past Paper Question – June 2023 – Paper 2 – Question 8 – Solution

8. (a) Complete the table for the function 𝑦 = −𝑥 2 + 𝑥 + 7. [2]

𝒙 −3 −2 −1 0 1 2 3 4

−5 1 5 7 7 5 1 −5
𝒚

When 𝑥 = −3, When 𝑥 = 1,

𝑦 = −(−3)2 + (−3) + 7 𝑦 = −(1)2 + (1) + 7

𝑦 = −9 − 3 + 7 𝑦 = −1 + 1 + 7

𝑦 = −5 𝑦=7

When 𝑥 = −1, When 𝑥 = 3,

𝑦 = −(−1)2 + (−1) + 7 𝑦 = −(3)2 + (3) + 7

𝑦 = −1 − 1 + 7 𝑦 = −9 + 3 + 7

𝑦=5 𝑦=1

217
(b) On the grid below, draw the graph of 𝑦 = −𝑥 2 + 𝑥 + 7 for −3 ≤ 𝑥 ≤ 4. [3]

× ×
𝑦 = −𝑥 2 + 𝑥 + 7

× ×

× ×

× ×

(c) Write down the coordinates of the maximum/minimum point of the

graph. [1]

7.2
0.5 , ……………)
(……………

218
(d) Write down the equation of the axis of symmetry of the graph. [1]

The equation of the axis of symmetry is 𝑥 = 0.5.

(e) Use your graph to find the solutions of the equation −𝑥 2 + 𝑥 + 7 = 0. [2]

−2.2
𝑥 = ………………………………………….. 3.2
or 𝑥 = …………………………………………..

(f) (i) On the grid on page 24, draw a line through the points (−3, −1) and

(0, 8). [1]

See graph above.

(ii) Determine the equation of this line in the form 𝑦 = 𝑚𝑥 + 𝑐. [2]

Points are (−3, −1) and (0, 8).

𝑦 −𝑦
Gradient, 𝑚 = 𝑥2−𝑥1
2 1

8−(−1)
Gradient, 𝑚 = 0−(−3)

8+1
Gradient, 𝑚 = 0+3

9
Gradient, 𝑚 = 3

Gradient, 𝑚 = 3

Substituting 𝑚 = 3 and point (0, 8) into 𝑦 − 𝑦1 = 𝑚(𝑥 − 𝑥1 ) gives:

219
𝑦 − 8 = 3(𝑥 − 0)

𝑦 − 8 = 3𝑥

𝑦 = 3𝑥 + 8

∴ The equation of this line is 𝑦 = 3𝑥 + 8.

Total: 12 marks

220
Section 8 – Geometry and Trigonometry

Objective 1: Explain concepts relating to geometry.

Geometric Concept Explanation

Points Points are stated in form (𝑥, 𝑦).

Lines A line is a straight path that extends infinitely in both

directions.

Parallel lines Parallel lines are two or more lines that never intersect.

They are always the same distance apart and have the same

slope.

Intersecting lines Intersecting lines are two or more lines that cross each

other at a point.

Perpendicular lines Perpendicular lines are two lines that intersect at a right

angle, which is a 90-degree angle.

Line segments A line segment is a part of a line that has two endpoints.

Rays A ray is a part of a line that has one endpoint and extends

infinitely in one direction. It is represented by a line with

one endpoint and an arrowhead to indicate the direction in

which it extends.

Curves A curve is a line that is not straight and may have varying

degrees of curvature.

Planes A plane is a flat, two-dimensional surface that extends

infinitely in all directions.

221
Types of angles There are several types of angles in geometry, including

acute angles (less than 90 degrees), right angles (exactly

90 degrees), obtuse angles (more than 90 degrees but less

than 180 degrees), straight angles (exactly 180 degrees),

and reflex angles (more than 180 degrees but less than 360

degrees).

Number of faces, edges, These are the three basic elements of three-dimensional

and vertices shapes. The faces are the flat surfaces of the shape, the

edges are the lines where two faces meet, and the vertices

are the points where three or more edges meet. For

example, a cube has six faces, 12 edges, and eight vertices.

222
Objective 2: Draw and measure angles and line segments accurately using appropriate

instruments.

Consider the diagram below:

𝑨 𝑩
parallel
𝑫 lines
𝑪

𝑭
𝑬
𝑯
𝑮

transversal

Here are the angles that are EQUAL:

1. Corresponding angles

• 𝐵, 𝐹 • 𝐷, 𝐻

• 𝐴, 𝐸 • 𝐶, 𝐺

2. Alternate angles

• 𝐵, 𝐺 • 𝐷, 𝐸

• 𝐴, 𝐻 • 𝐶, 𝐹

223
3. Vertically Opposite angles

• 𝐵, 𝐶 • 𝐴, 𝐷

• 𝐹, 𝐺 • 𝐸, 𝐻

Here are the angles that ADD UP TO 𝟏𝟖𝟎°:

1. Co-interior angles

• 𝐷+𝐹

• 𝐶+𝐸

2. Supplementary angles

• 𝐴+𝐵 • 𝐸+𝐹

• 𝐶+𝐷 • 𝐺+𝐻

• 𝐴+𝐶 • 𝐸+𝐺

• 𝐵+𝐷 • 𝐹+𝐻

224
Objective 3: Construct lines, angles, and polygons using appropriate instruments.

You should be aware of:

1. Parallel and perpendicular lines

2. Bisecting line segments and angles

3. Constructing a line perpendicular to another line, 𝑙, from a point that is not on the

line, 𝑙.

4. Triangles, quadrilaterals, regular and irregular polygons

5. Angles include 30°, 45°, 60°, 90°, 120° and their combinations.

225
Question:

Using a pencil, ruler and a pair of compasses, construct a triangle 𝑋𝑌𝑍 such that

𝑋𝑌 = 7.5 𝑐𝑚, ∠𝑋𝑌𝑍 = 120° and 𝑌𝑍 = 5.5 𝑐𝑚.

Solution:

226
Objective 4: Identify the type(s) of symmetry possessed by a given plane figure.

Line of symmetry

• A line of symmetry is a line that divides a shape into two congruent halves that are

mirror images of each other.

• For example, a square has four lines of symmetry that bisect each pair of opposite

sides, while a scalene triangle has no lines of symmetry.

Rotational symmetry

• Rotational symmetry is a type of symmetry that occurs when a shape can be

rotated by a certain angle around a fixed point, called the center of rotation, and

still look the same.

• For example, a regular pentagon has rotational symmetry of order 5, because it

can be rotated by 72 degrees around its center and still look the same.

Order of rotational symmetry

• The order of rotational symmetry is the number of times a shape can be rotated

by its angle of rotational symmetry and still look the same.

• For example, a circle has rotational symmetry of infinite order, because it looks

the same no matter how many times it is rotated around its center.

227
Objective 5: Solve geometric problems using properties of:

(a) lines, angles, and polygons

(b) congruent triangles

(c) similar figures

(d) faces, edges and vertices of solids

(e) classes of solids

You should be aware of:

1. Determining and justifying the measure of angles: adjacent angles, angles at a

point, supplementary angles, complementary angles, vertically opposite angles.

2. Parallel lines and transversals, alternate angles, corresponding angles, co-interior

angles.

3. Triangles: equilateral, isosceles, scalene, obtuse, right, acute.

4. Quadrilaterals: Square, rectangle, rhombus, kite, parallelogram, trapezium

5. Cases of congruency.

6. Properties of similar triangles

228
Square:

The properties of a square are:

• Opposite sides are parallel


• All sides are equal
• All angles are right angles (90°)

Lines of symmetry: 4

229
Rectangle:

V
V V

The properties of a rectangle are:

• Opposite sides are equal and parallel


• All angles are right angles (90°)

Lines of symmetry: 2

230
Rhombus:

The properties of a rhombus are:

• Opposite sides are parallel


• All sides are equal
• Opposite angles are equal
• Diagonals bisect each other at right angles (90°)

Lines of symmetry: 2

231
Kite:

The properties of a kite are:

• Two pairs of adjacent sides are equal


• One pair of opposite sides are equal
• One diagonal bisects the other
• Diagonals intersect at right angles (90°)

Lines of symmetry: 1

232
Parallelogram:

V
V

The properties of a parallelogram are:

• Opposite sides are equal and parallel


• Opposite angles are equal

Lines of symmetry: 0

233
Trapezoid:

V
V

The property of a trapezoid is:

• One pair of opposite sides is parallel

Lines of symmetry: 1

234
Definitions:

• Supplementary angles add up to 180°.

• Complementary angles add up to 90°.

Consider the diagram below:

Angle → degree of turn

Types of angles include:

1. acute angle – less than 90°

2. obtuse angle – more than 90° but less than 180°

3. reflex angle – more than 180° but less than 360°

reflex

obtuse
acute

Note:

• Angles are measured in degrees.

• Ensure that your calculator is in “degree” mode. It should have a “D” at the top.

235
Objective 6: Solve geometric problems using properties of circles and circle theorems.

Recall the following terms:

Term Diagram Note

arc An arc is a piece of

circumference.

chord A chord is a straight line

connecting two points on


𝐵
the circumference. For

example, 𝐴𝐵 is a chord.

segment A segment is a part of a

circle bounded by a chord


𝑃
and an arc. For example, 𝑃

𝑄 is the major segment and 𝑄

is the minor segment.

sector A sector is bounded by two

radii and an arc. For


𝑃
example, 𝑃 is the major

sector and 𝑄 is the minor


𝑄
sector.

236
tangent A tangent touches the circle

once at its circumference.

normal

A normal to a circle is a

straight line drawn at 90

degrees to the tangent.

tangent

Circle Theorems

Theorem 1: The angle formed in a semicircle is always a right angle.

237
Theorem 2: The angle at the centre of a circle is twice the angle at the circumference of

the circle from the same chord.

𝑎°

2𝑎°

Theorem 3: Angles from the same chord in the same segment are equal.

𝑎°
𝑎°

𝑏°

𝑏°
chord

238
Theorem 4: Opposite angles of a cyclic quadrilateral add up to 180°, that is, 𝑎 + 𝑐 = 180°

and 𝑏 + 𝑑 = 180°.

𝑐°

𝑏° Note: A quadrilateral
is a figure with 4 sides.
A cyclic quadrilateral
is one where all the
vertices touch the
circumference of the
𝑎° circle.
𝑑°

Theorem 5: The perpendicular line from the centre of a circle to a chord, bisects the chord.

239
Theorem 6: Two radii will give you an isosceles triangle.

Theorem 7: The angle between a tangent and the radius, at the point where the tangent

touches the circle, is a right angle.

240
Theorem 8: Two tangents drawn from a point to a circle are equal.

Theorem 9: The angle between a tangent and a chord is equal to the angle at the

circumference in the alternate segment.

𝑏°
𝑎° Note: This circle
theorem is called the
Alternate Segment
Theorem.

𝑎° 𝑏°

241
Question:

Consider the diagram below:

𝑂
𝑏

𝑐
𝑀 𝐾

154°
𝑁

Find the values of 𝑎, 𝑏 and 𝑐.

Solution:

Opposite angles in a cyclic quadrilateral add up to 180°.

𝑎 = 180° − 154°

𝑎 = 26°

The angle at the centre is twice the angle at the circumference, standing on the same

chord.

𝑏 = 2(26°)

𝑏 = 52°

𝑂𝑀 and 𝑂𝐾 are both radii of the same circle and are therefore, equal to each other.

Angles in a triangle add up to 180°.

242
180°−52°
𝑐= 2

128°
𝑐= 2

𝑐 = 64°

243
Objective 7: Represent translations in a plane using vectors.

Objective 8: Determine and represent the location of:

(a) the image of an object under a transformation

(b) an object given the image under a transformation

Translation or Slide

𝑥
Use the vector: (𝑦)

where 𝑥 represents the movement in the horizontal

and 𝑦 represents the vertical movement.

Consider the graph below.

𝐴
×

×
𝐴′

244
Translation from 𝑨 → 𝑨′:

Across = 3

Vertical = −5

3
∴ The translation from 𝐴 → 𝐴′ is ( ).
−5

245
Objective 9: State the relationship between an object and its image in the plane under

geometric transformations.

Objective 10: Describe a transformation given an object and its image.

Objective 11: Locate the image of an object under a combination of transformations.

Reflection or Flip

𝑥-axis 𝑦-axis

1 0 −1 0
( ) ( )
0 −1 0 1

The line 𝑦 = 𝑥 The line 𝑦 = −𝑥

0 1 0 −1
( ) ( )
1 0 −1 0

Translation or Slide

𝑥
Use the vector: (𝑦)

where 𝑥 represents the movement in the horizontal

and 𝑦 represents the vertical movement.

Common Rotations

Rotation 90 degrees clockwise (or 270 degrees anticlockwise):

0 1
( )
−1 0

Rotation 180 degrees:

−1 0
( )
0 −1

246
Rotation 270 degrees clockwise (or 90 degrees anticlockwise):

0 −1
( )
1 0

Enlargement

2 0
An enlargement of scale factor 2 is ( ).
0 2

247
Objective 12: Use Pythagoras’ theorem to solve problems.

Pythagoras’ Theorem - can be used to find a missing side given the other two sides.

It states that the square of the hypotenuse is equal to the sum of the square of the other

two sides.

𝑐
𝑏

𝒄𝟐 = 𝒂𝟐 + 𝒃𝟐

𝒄 = √𝒂𝟐 + 𝒃𝟐

248
Objective 13: Define the trigonometric ratios of acute angles in a right triangle.

Objective 14: Relate objects in the physical world to geometric objects.

Objective 15: Apply the trigonometric ratios to solve problems.

To compute the sine, cosine and tangent of an angle, the following formulas are used:

𝒐𝑝𝑝
• 𝐬in 𝜃 = 𝒉𝑦𝑝

𝒂𝑑𝑗
• 𝐜os 𝜃 = 𝒉𝑦𝑝

𝒐𝑝𝑝
• 𝐭an 𝜃 = 𝒂𝑑𝑗

To remember these formulas, you can just remember the first letter of each part.

Soh = Sine is Opposite over Hypotenuse.

Cah = Cosine is Adjacent over Hypotenuse.

Toa = Tangent is Opposite over Adjacent.

249
Objective 16: Use the sine and cosine rules to solve problems involving triangles.

Sine rule:

𝑏 𝐵

𝒂 𝒃 𝒄 𝐬𝐢𝐧 𝑨 𝐬𝐢𝐧 𝑩 𝐬𝐢𝐧 𝑪


= 𝐬𝐢𝐧 𝑩 = 𝐬𝐢𝐧 𝑪 OR = =
𝐬𝐢𝐧 𝑨 𝒂 𝒃 𝒄

Cosine rule:

𝑏
𝐵

𝑎
𝐶

𝒂𝟐 = 𝒃𝟐 + 𝒄𝟐 − 𝟐𝒃𝒄 𝐜𝐨𝐬 𝑨

250
Question 1:

Consider the diagram below and find the value of 𝑎.

𝐴
𝑐
110° 70 𝑘𝑚
𝑏
50 𝑘𝑚
𝐵

𝑎
𝐶

Solution:

Using the cosine rule,

𝑎2 = 𝑏 2 + 𝑐 2 − 2𝑏𝑐 cos 𝐴

𝑎2 = (50)2 + (70)2 − 2(50)(70) cos 110°

𝑎2 = 2500 + 4900 − 7000 cos 110°

𝑎2 = 9794.141003

𝑎 = √9794.141003

𝑎 = 98.97 𝑘𝑚 (to 2 decimal places)

251
Question 2:

Find the value of 𝜃.

15°
𝐵
14
𝑎

Solution:

𝑎 𝑏
= sin 𝐵
sin 𝐴

sin 𝜃 sin 15
=
14 4

sin 15
sin 𝜃 = × 14
4

sin 𝜃 = 0.90586665

𝜃 = sin−1(0.90586665)

𝜃 = 64.9° (to 1 decimal place)

252
Objective 17: Solve problems involving bearings

In mathematics, a bearing is the angle in degrees measured clockwise from North. In

bearings, you need to know the following:

➢ Cardinal Points

➢ Know your North – “draw in north if you have to.”

➢ Use extensions

➢ Use three figures when representing the size of the angle, for example, 030°.

Representation of Bearings

𝐵 𝐵

𝐴 𝐴

Bearing of 𝑨 from 𝑩 Bearing of 𝑨 to 𝑩

253
Past Paper Question – June 2023 – Paper 2 – Question 9

9. (a) 𝐿, 𝑀, 𝑁 and 𝑅 are points on the circumference of a circle, with centre 𝑂. 𝑃𝑄 is

a tangent to the circle at 𝑅. Angle 𝑃𝑅𝐿 = 48° and Angle 𝑅𝑂𝑁 = 156°.

𝑵 𝑸

𝑴 𝒂 𝒆

𝑹
𝟒𝟖°
𝒓
𝑳

Find the value of EACH of the following angles, giving reasons for EACH of your

answers. Show ALL working where appropriate.

(i) Angle 𝑟 [2]

(ii) Angle 𝑒 [2]

(iii) Angle 𝑎 [2]

254
(b) The diagram below shows a triangular field, 𝐿𝑀𝑃, on horizontal ground.

North, 𝑵

𝟏𝟐𝟎 𝒎

𝟏𝟓𝟎 𝒎

𝟏𝟖𝟎 𝒎
𝑷

(i) Calculate the value of Angle 𝑀𝐿𝑃. [3]

(ii) The bearing of 𝑃 from 𝐿 is 210°.

(a) Find the bearing of 𝑀 from 𝐿. [1]

(b) Calculate the value of Angle 𝑁𝐿𝑃 and hence, find the bearing of 𝐿

from 𝑃. [2]

Total: 12 marks

255
Past Paper Question – June 2023 – Paper 2 – Question 9 – Solution

9. (a) 𝐿, 𝑀, 𝑁 and 𝑅 are points on the circumference of a circle, with centre 𝑂. 𝑃𝑄 is

a tangent to the circle at 𝑅. Angle 𝑃𝑅𝐿 = 48° and Angle 𝑅𝑂𝑁 = 156°.

𝑵 𝑸

𝑴 𝒂 𝒆
𝒙
𝑹
𝟒𝟖°
𝒓
𝑳

Find the value of EACH of the following angles, giving reasons for EACH of your

answers. Show ALL working where appropriate.

(i) Angle 𝑟 [2]

The angle at the centre of the circle is twice the angle at the

circumference of the circle standing on the same chord.

Angle 𝑟 = 156° ÷ 2

Angle 𝑟 = 78°

256
(ii) Angle 𝑒 [2]

Angle 𝑒 is the angle that the chord 𝑁𝑅 makes with the tangent 𝑃𝑄.

So, angle 𝑒 is equal to the angle in the alternate segment, Angle 𝑟.

∴ Angle 𝑒 = 78°

(iii) Angle 𝑎 [2]

The angles at 𝑅 are supplementary and add up to 180°.

Angle 𝑥 + Angle 𝑒 + 48° = 180°

Angle 𝑥 + 78° + 48° = 180°

Angle 𝑥 = 180° − 78° − 48°

Angle 𝑥 = 54°

Opposite angles in a cyclic quadrilateral add up to 180°.

Angle 𝑎 + Angle 𝑥 = 180°

Angle 𝑎 + 54° = 180°

Angle 𝑎 = 180° − 54°

Angle 𝑎 = 126°

257
(b) The diagram below shows a triangular field, 𝐿𝑀𝑃, on horizontal ground.

North, 𝑵

𝟏𝟐𝟎 𝒎

𝟏𝟓𝟎 𝒎

𝟏𝟖𝟎 𝒎
𝑷

(i) Calculate the value of Angle 𝑀𝐿𝑃. [3]

Using the cosine rule,

(𝑃𝑀)2 = (𝑃𝐿)2 + (𝐿𝑀)2 − 2(𝑃𝐿)(𝐿𝑀) cos 𝑀𝐿̂𝑃

(180)2 = (150)2 + (120)2 − 2(150)(120) cos 𝑀𝐿̂𝑃

32400 = 22500 + 14400 − 36000 cos 𝑀𝐿̂𝑃

32400 − 22500 − 14400 = −36000 cos 𝑀𝐿̂ 𝑃

−4500 = −36000 cos 𝑀𝐿̂ 𝑃

−4500
cos 𝑀𝐿̂𝑃 = −36000

1
cos 𝑀𝐿̂𝑃 = 8

1
𝑀𝐿̂𝑃 = cos−1 (8)

𝑀𝐿̂𝑃 = 82.8° (to 1 decimal place)

258
(ii) The bearing of 𝑃 from 𝐿 is 210°.

(a) Find the bearing of 𝑀 from 𝐿. [1]

Consider:

North, 𝑵

𝑳 𝟐𝟏𝟎°

𝟖𝟐. 𝟖°

Bearing of 𝑀 from 𝐿 = 210° − 82.8°

Bearing of 𝑀 from 𝐿 = 127.2°

(b) Calculate the value of Angle 𝑁𝐿𝑃 and hence, find the bearing of 𝐿

from 𝑃. [2]

Angle 𝑁𝐿𝑃 = 360° − 210°

Angle 𝑁𝐿𝑃 = 150°

Since Angle 𝑁𝐿𝑃 and the bearing of 𝐿 from 𝑃 are co-interior angles,

Bearing of 𝐿 from 𝑃 = 180° − 150°

Bearing of 𝐿 from 𝑃 = 30°

Total: 12 marks

259
Section 9 – Vectors and Matrices

Objective 1: Explain concepts associated with vectors.

Concept of a vector:

➢ A vector is a quantity that has magnitude as well as direction.

Consider the coordinate (4, 3).

𝟒
The coordinate represented as a vector is ( ).
𝟑

where 4 tells us : How much across?

and 3 tells us : How much up/down?

The vector is represented below:

4
The vector is ⃗⃗⃗⃗⃗
𝑃𝑄 = ( ).
3

4
We can use a letter to represent the vector. Hence, 𝒂 = ( ).
3

260
Position Vectors

Consider the point 𝐴(𝑥, 𝑦) in the three-dimensional space.

⃗⃗⃗⃗⃗
𝑂𝐴 = 𝑥𝒊̂ + 𝑦𝒋̂

𝑥
⃗⃗⃗⃗⃗ = ( )
𝑂𝐴 𝑦

⃗⃗⃗⃗⃗ is a position vector.


Vector 𝑂𝐴

A position vector is a vector relative to the origin.

Displacement Vectors

Consider the position vectors ⃗⃗⃗⃗⃗ ⃗⃗⃗⃗⃗ below.


𝑂𝐴 and 𝑂𝐵

𝑥
𝑂

The displacement vector ⃗⃗⃗⃗⃗


𝐴𝐵 is given by:

⃗⃗⃗⃗⃗⃗
𝑨𝑩 = ⃗⃗⃗⃗⃗⃗
𝑨𝑶 + ⃗⃗⃗⃗⃗⃗
𝑶𝑩

⃗⃗⃗⃗⃗ = −𝑂𝐴
𝐴𝐵 ⃗⃗⃗⃗⃗ + 𝑂𝐵
⃗⃗⃗⃗⃗

⃗⃗⃗⃗⃗⃗
𝑨𝑩 = ⃗⃗⃗⃗⃗⃗
𝑶𝑩 − ⃗⃗⃗⃗⃗⃗
𝑶𝑨

261
Parallel vectors

Two vectors are said to be parallel if we can obtain one from the other by multiplying it

by a scalar.

1 2
1. Consider 𝒂 = ( ) and 𝒃 = ( ).
2 4

𝒂 and 𝒃 are parallel vectors since 𝒃 = 2𝒂.

2. Consider ⃗⃗⃗⃗⃗ ⃗⃗⃗⃗⃗ = 2𝒖 + 6𝒗.


𝑃𝑄 = 𝒖 + 3𝒗 and 𝑄𝑅

⃗⃗⃗⃗⃗ and 𝑄𝑅
𝑃𝑄 ⃗⃗⃗⃗⃗ are parallel since 𝑄𝑅
⃗⃗⃗⃗⃗ = 2𝑃𝑄
⃗⃗⃗⃗⃗ .

Colinear vectors

Colinear vectors are parallel and can pass through a same point.

Equal vectors

Two vectors are equal if they have the same magnitude and direction.

Example:

2𝑎 6
Given that ( ) = ( ), find the value of 𝑎.
0 0

Solution:

Equating corresponding entries gives:

2𝑎 = 6
6
𝑎=2

𝑎=3

262
Inverse vectors

Two vectors are said to be inverse or opposite if they have the same magnitude but

opposite directions.

1 −1
For example, ( ) and ( ) are inverse vectors.
2 −2

263
Objective 2: Simplify expressions involving vectors.

Addition of vectors

Vectors of the same size can be added by adding the corresponding entries.

𝑢1 𝑢2 𝑢 +𝑢
( 𝜈 ) + ( 𝜈 ) = ( 𝜈1 + 𝑣 2 )
1 2 1 2

Subtraction of vectors

Vectors of the same size can be subtracted by subtracting the corresponding entries.

𝑢1 𝑢2 𝑢1 − 𝑢2
(𝜈 ) − (𝜈 ) = ( 𝜈 − 𝑣 )
1 2 1 2

Scalar multiplication of vectors

When a vector is multiplied by a scalar, 𝜆, each component is multiplied by the scalar.

𝑢1 𝜆𝑢
𝜆 (𝑢 ) = ( 1 )
2 𝜆𝑢2

Example 1:

4 8
2( )=( )
−3 −6

Example 2:

5
1 5
( ) = (2)
2 10 5

264
⃗⃗⃗⃗⃗ = (𝑎 ) where 𝑂 is the origin
Objective 3: Write the position vector of a point 𝑃(𝑎, 𝑏) as 𝑂𝑃
𝑏

(0, 0)

Example:

Consider the point 𝐴 = (2, 3).

2
Then, the position vector, ⃗⃗⃗⃗⃗
𝑂𝐴 = ( )
3

265
Objective 4: Determine the magnitude of a vector.

The magnitude or size of a vector can be found using:

⃗⃗⃗⃗⃗⃗ | = √𝒙𝟐 + 𝒚𝟐
|𝑨𝑩

Note that the magnitude of a unit vector is 1.

Question:

5
Find the magnitude of ⃗⃗⃗⃗⃗
𝐴𝐵 = ( ).
−12

Solution:

⃗⃗⃗⃗⃗ | = √(5)2 + (−12)2


|𝐴𝐵

⃗⃗⃗⃗⃗ | = √25 + 144


|𝐴𝐵

⃗⃗⃗⃗⃗ | = √169
|𝐴𝐵

⃗⃗⃗⃗⃗ | = 13 units
|𝐴𝐵

266
Objective 5: Determine the direction of a vector.

Objective 6: Use vectors to solve problems in geometry.

Example:

𝑦
𝐵

5𝑁

30°
𝐴

The magnitude of the vector is 5 𝑁.

The direction of the vector is 30° (anticlockwise) from the horizontal.

⃗⃗⃗⃗⃗ = 5 𝑁, 0° (anticlockwise) from the horizontal.


∴ 𝐴𝐵

267
Past Paper Question – January 2009 – Paper 2 – Question 13

The diagram below shows position vectors ⃗⃗⃗⃗⃗ ⃗⃗⃗⃗⃗⃗ .


𝑂𝑃 and 𝑂𝑄

𝑥
(a) Write as a column vector, in the form (𝑦)

(i) ⃗⃗⃗⃗⃗
𝑂𝑃 [1]

(ii) ⃗⃗⃗⃗⃗⃗
𝑂𝑄 [1]

(b) The point 𝑅 has coordinates (8, 9).

𝑥
(i) ⃗⃗⃗⃗⃗ as a vector in the form ( ).
Express 𝑄𝑅 [2]
𝑦

(ii) Using a vector method, show that ⃗⃗⃗⃗⃗ ⃗⃗⃗⃗⃗ .


𝑂𝑃 is parallel to 𝑄𝑅 [1]

(iii) ⃗⃗⃗⃗⃗ .
Determine the magnitude of the vector 𝑃𝑅 [2]

(c) The point 𝑆 has coordinates (𝑎, 𝑏).

(i) ⃗⃗⃗⃗⃗ as a column vector, in terms of 𝑎 and 𝑏.


Write 𝑄𝑆 [2]

(ii) Given that ⃗⃗⃗⃗⃗


𝑄𝑆 = ⃗⃗⃗⃗⃗
𝑂𝑃 , calculate the value of 𝑎 and the value of 𝑏. [3]

(iii) Using a vector method, show that 𝑂𝑃𝑆𝑄 is a parallelogram. [3]

Total: 15 marks

268
Past Paper Question – January 2009 – Paper 2 – Question 13 – Solution

The diagram below shows position vectors ⃗⃗⃗⃗⃗ ⃗⃗⃗⃗⃗⃗ .


𝑂𝑃 and 𝑂𝑄

𝑥
(a) Write as a column vector, in the form (𝑦)

(i) ⃗⃗⃗⃗⃗
𝑂𝑃 [1]

The coordinates of 𝑃 = (3, 2).

3
The vector ⃗⃗⃗⃗⃗
𝑂𝑃 = ( ).
2

(ii) ⃗⃗⃗⃗⃗⃗
𝑂𝑄 [1]

The coordinates of 𝑄 = (−1, 3).

⃗⃗⃗⃗⃗⃗ = (−1).
The vector 𝑂𝑄
3

269
(b) The point 𝑅 has coordinates (8, 9).

𝑥
(i) ⃗⃗⃗⃗⃗ as a vector in the form ( ).
Express 𝑄𝑅 [2]
𝑦

⃗⃗⃗⃗⃗ = (8).
Since 𝑅 has coordinates (8, 9), then 𝑂𝑅
9

Using the triangle law,

⃗⃗⃗⃗⃗ = 𝑂𝑅
𝑄𝑅 ⃗⃗⃗⃗⃗ − 𝑂𝑄
⃗⃗⃗⃗⃗⃗

⃗⃗⃗⃗⃗ = (8) − (−1)


𝑄𝑅
9 3

⃗⃗⃗⃗⃗ = (9)
𝑄𝑅
6

(ii) ⃗⃗⃗⃗⃗ is parallel to 𝑄𝑅


Using a vector method, show that 𝑂𝑃 ⃗⃗⃗⃗⃗ . [1]

⃗⃗⃗⃗⃗ = 3𝑂𝑃
𝑄𝑅 ⃗⃗⃗⃗⃗

The vectors are related by a scalar factor of 3.

Therefore, they are parallel.

(iii) Determine the magnitude of the vector ⃗⃗⃗⃗⃗


𝑃𝑅 . [2]

Using the triangle law,

⃗⃗⃗⃗⃗ = 𝑂𝑅
𝑃𝑅 ⃗⃗⃗⃗⃗ − 𝑂𝑃
⃗⃗⃗⃗⃗

⃗⃗⃗⃗⃗ = (8) − (3)


𝑃𝑅
9 2

⃗⃗⃗⃗⃗ 5
𝑃𝑅 = ( )
7

270
⃗⃗⃗⃗⃗ | = √(5)2 + (7)2
|𝑃𝑅

⃗⃗⃗⃗⃗ | = √25 + 49
|𝑃𝑅

⃗⃗⃗⃗⃗ | = √74
|𝑃𝑅

⃗⃗⃗⃗⃗ | = 8.60 units


|𝑃𝑅 (to 3 significant figures)

(c) The point 𝑆 has coordinates (𝑎, 𝑏).

(i) Write ⃗⃗⃗⃗⃗


𝑄𝑆 as a column vector, in terms of 𝑎 and 𝑏. [2]

𝑎
Since 𝑆 has coordinates (𝑎, 𝑏), then ⃗⃗⃗⃗⃗
𝑂𝑆 = ( ).
𝑏

Using the triangle law,

⃗⃗⃗⃗⃗ = 𝑂𝑆
𝑄𝑆 ⃗⃗⃗⃗⃗ − 𝑂𝑄
⃗⃗⃗⃗⃗⃗

⃗⃗⃗⃗⃗ = (𝑎) − (−1)


𝑄𝑆
𝑏 3

⃗⃗⃗⃗⃗ 𝑎+1
𝑄𝑆 = ( )
𝑏−3

(ii) Given that ⃗⃗⃗⃗⃗


𝑄𝑆 = ⃗⃗⃗⃗⃗
𝑂𝑃 , calculate the value of 𝑎 and the value of 𝑏. [3]

⃗⃗⃗⃗⃗
𝑄𝑆 = ⃗⃗⃗⃗⃗
𝑂𝑃

𝑎+1 3
( )=( )
𝑏−3 2

Equating corresponding entries gives:

𝑎+1=3 and 𝑏−3=2

𝑎 = 3−1 𝑏 = 2+3

𝑎=2 𝑏=5

271
(iii) Using a vector method, show that 𝑂𝑃𝑆𝑄 is a parallelogram. [3]

⃗⃗⃗⃗⃗ 3 3
𝑂𝑃 = ( ) and ⃗⃗⃗⃗⃗
𝑄𝑆 = ( )
2 2

Consider the diagram below:

𝑷 𝑺

𝑶 𝑸

Now, ⃗⃗⃗⃗ ⃗⃗⃗⃗⃗⃗ and ⃗⃗⃗⃗⃗


𝑃𝑆 = 𝑂𝑄 𝑂𝑃 = ⃗⃗⃗⃗⃗
𝑄𝑆.

Since one pair of opposite sides of the quadrilateral is both parallel and

equal, then the quadrilateral is a parallelogram.

Hence, 𝑂𝑃𝑆𝑄 is a parallelogram.


Q.E.D.

Total: 15 marks

272
Past Paper Question – June 2023 – Paper 2 – Question 10(c)

10. (c) The following vectors are defined as shown below.

⃗⃗⃗⃗⃗⃗⃗ = ( 5 )
𝑊𝑋 ⃗⃗⃗⃗⃗ = (−3)
𝑋𝑌 ⃗⃗⃗⃗⃗ 8
𝑍𝑌 = ( )
−1 7 −7

Determine EACH of the following.

5
(i) A vector, other than ( ), that is parallel to ⃗⃗⃗⃗⃗⃗⃗
𝑊𝑋 [1]
−1

(ii) ⃗⃗⃗⃗⃗⃗⃗
𝑊𝑌 [1]

(iii) ⃗⃗⃗⃗⃗
𝑋𝑍 [2]

(iv) ⃗⃗⃗⃗⃗ |
|𝑋𝑌 [2]

273
Past Paper Question – June 2023 – Paper 2 – Question 10(c) – Solution

10. (c) The following vectors are defined as shown below.

⃗⃗⃗⃗⃗⃗⃗ = ( 5 )
𝑊𝑋 ⃗⃗⃗⃗⃗ = (−3)
𝑋𝑌 ⃗⃗⃗⃗⃗ 8
𝑍𝑌 = ( )
−1 7 −7

Determine EACH of the following.

5
(i) A vector, other than ( ), that is parallel to ⃗⃗⃗⃗⃗⃗⃗
𝑊𝑋 [1]
−1

⃗⃗⃗⃗⃗⃗⃗ is ( 10 10 5
A vector parallel to 𝑊𝑋 ) since ( ) = 2 ( ).
−2 −2 −1

(ii) ⃗⃗⃗⃗⃗⃗⃗
𝑊𝑌 [1]

⃗⃗⃗⃗⃗⃗⃗ = 𝑊𝑋
𝑊𝑌 ⃗⃗⃗⃗⃗⃗⃗ + 𝑋𝑌
⃗⃗⃗⃗⃗

⃗⃗⃗⃗⃗⃗⃗ 5 −3
𝑊𝑌 = ( ) + ( )
−1 7

⃗⃗⃗⃗⃗⃗⃗ = (5 + (−3))
𝑊𝑌
−1 + 7

⃗⃗⃗⃗⃗⃗⃗ 2
𝑊𝑌 = ( )
6

(iii) ⃗⃗⃗⃗⃗
𝑋𝑍 [2]

⃗⃗⃗⃗⃗
𝑋𝑍 = ⃗⃗⃗⃗⃗
𝑋𝑌 + ⃗⃗⃗⃗⃗
𝑌𝑍

⃗⃗⃗⃗⃗ = 𝑋𝑌
𝑋𝑍 ⃗⃗⃗⃗⃗ − ⃗⃗⃗⃗⃗
𝑍𝑌

⃗⃗⃗⃗⃗ −3 8
𝑋𝑍 = ( ) − ( )
7 −7

⃗⃗⃗⃗⃗ = ( −3 − 8 )
𝑋𝑍
7 − (−7)

⃗⃗⃗⃗⃗ = (−11)
𝑋𝑍
14
274
(iv) ⃗⃗⃗⃗⃗ |
|𝑋𝑌 [2]

⃗⃗⃗⃗⃗ | = √(−3)2 + (7)2


|𝑋𝑌

⃗⃗⃗⃗⃗ | = √9 + 49
|𝑋𝑌

⃗⃗⃗⃗⃗ | = √58
|𝑋𝑌

⃗⃗⃗⃗⃗ | = 7.62 units


|𝑋𝑌 (to 3 significant figures)

Total: 12 marks

275
Matrices

Objective 7: Explain basic concepts associated with matrices.

3 4
Consider the matrix ( ).
2 𝑥

The elements or members of the matrix are 3, 4, 2 and 𝑥.

row

column

So, elements of the first row are 3 and 4.

The size or order of a matrix is stated in the form: rows by column

2
( )
4

The size is 2 × 1.

3 2
( )
4 1

The size is 2 × 2. This is a square matrix as it has the same number of rows and columns.

1 0
The 2 × 2 identity matrix is 𝐼 = ( ).
0 1

Note that if a matrix 𝐴 is multiplied by the identity matrix, the result is matrix 𝐴.

𝑨×𝑰= 𝑰×𝑨= 𝑨

276
Objective 8: Solve problems involving matrix operations.

Addition and Subtraction of Matrices


2 5 1
Consider ( ) + ( ).
3 2 0

This is invalid! The matrices are not of the same size. Therefore, it cannot be added
together.

NB: In order to add, matrices must be of the same order/size.

Equivalent Matrices

Example:

2𝑎 −1 −20 −1
( )=( )
4 𝑏 4 6

Find the value of a and b.

Solution:

2𝑎 = −20

−20
𝑎= 2

𝑎 = −10

𝑏=6

Multiplication in Matrices

There are two subtopics: 1. Scalar Multiplication

2. Matrix Multiplication
277
Scalar Multiplication

Scalar multiplication involves a matrix being multiplied by a number.

It is of the form 𝒂(Matrix) where 𝑎 is a number or constant.

Example:
4 6
Evaluate 3 ( ).
2 −1

Solution:

4 6 4×3 6×3
3( )=( )
2 −1 2 × 3 −1 × 3

4 6 12 18
3( )=( )
2 −1 6 −3

Matrix Multiplication

( )( )
r×c r×c

The number of columns in the first matrix must be equal to the number of rows in the

second matrix.

For example,

3 6 1
( ) ( )
4 −1 2

2×2 2×1

We can multiply these two matrices.

278
However,

1 3 6
( ) ( )
2 4 −1

2×1 2×2

We cannot multiply these two matrices.

Note: Matrix multiplication is NOT commutative.

CXC has two styles:


2 3 1
Style 1: ( )( ) 2 by 2 × 2 by 1
1 0 4

2 3 1 −2
Style 2: ( )( ) 2 by 2 × 2 by 2
1 0 5 10

Example:

3 −1 2
Evaluate ( ) ( ).
10 0 4

Solution:

3 −1 2
( )( )
10 0 4

(3 × 2) + (−1 × 4)
=( )
(10 × 2) + (0 × 4)

6 + (−4)
=( )
20 + 0

2
=( )
20

279
Example:
4 1 −3 0
Evaluate ( )( ).
0 2 −1 2

Solution:

4 1 −3 0
=( )( )
0 2 −1 2

(4 × −3) + (1 × −1) (4 × 0) + (1 × 2)
=( )
(0 × −3) + (2 × −1) (0 × 0) + (2 × 2)

−12 + (−1) 0 + 2
=( )
0 + (−2) 0+4

−13 2
=( )
−2 4

280
Objective 9: Evaluate the determinant of a ‘2 × 2’ matrix.

𝑎 𝑏
Suppose 𝑀 = ( ).
𝑐 𝑑

𝐝𝐞𝐭(𝑴) = 𝒂𝒅 − 𝒃𝒄

281
Objective 10: Define the multiplicative inverse of a non-singular square matrix.

The multiplicative inverse of a non-singular square matrix A is a matrix 𝐴−1 , such that the

product of 𝐴 and 𝐴−1 is the identity matrix 𝐼.

𝑨 × 𝑨−𝟏 = 𝑰

Note:

The existence of a multiplicative inverse of a non-singular square matrix A depends on

the determinant of A. If det(A) is non-zero, then A has a unique multiplicative inverse. If

det(A) is zero, then A does not have a multiplicative inverse, and is said to be singular.

282
Objective 11: Obtain the inverse of a non-singular ‘2 × 2’ matrix.

𝑎 𝑏
Suppose 𝑀 = ( ).
𝑐 𝑑

The adjoint of the matrix 𝑀 is:

𝒅 −𝒃
𝒂𝒅𝒋(𝑴) = ( )
−𝒄 𝒂

The inverse of the matrix 𝑀 is:

𝟏
𝑴−𝟏 = 𝐝𝐞𝐭(𝑴) × 𝒂𝒅𝒋(𝑴)

283
Objective 12: Determine a ‘2 × 2’ matrix associated with a specified transformation.

Objective 13: Use matrices to solve simple problems in Arithmetic, Algebra and

Geometry.

Tip:

1. If given two matrices 𝑆 and 𝑅 and you are asked to find the combined

transformation 𝑆 followed by 𝑅, this means 𝑅𝑆.

284
Past Paper Question – January 2009 – Paper 2 – Question 14
1 2 1 3
(a) Calculate the matrix product 3𝐴𝐵, where 𝐴 = ( ) and 𝐵 = ( ). [3]
2 1 2 5

(b) The diagram below, not drawn to scale, shows a triangle, 𝐴𝐵𝐶 whose coordinates are

stated.

Triangle 𝐴𝐵𝐶 undergoes two successive transformations, 𝑉 followed by 𝑊, where

2 0 −1 0
𝑉=( ) and 𝑊 = ( ).
0 2 0 1

(i) State the effect of 𝑉 on triangle 𝐴𝐵𝐶. [2]

(ii) Determine the 2 × 2 matrix that represents the combined transformation of 𝑉

followed by 𝑊. [3]

(iii) Use your matrix in (b)(ii), determine the coordinates of the image of triangle

𝐴𝐵𝐶 under this combined transformation. [3]

(c) (i) Write the following simultaneous equations in the form 𝐴𝑋 = 𝐵 where 𝐴, 𝑋 and 𝐵

are matrices:

11𝑥 + 6𝑦 = 6

9𝑥 + 5𝑦 = 7 [2]

285
(ii) Hence, write the solution for 𝑥 and 𝑦 as a product of two matrices. [2]

Total: 15 marks

286
January 2009 – Paper 2 – Question 14 - Solution
1 2 1 3
(a) Calculate the matrix product 3𝐴𝐵, where 𝐴 = ( ) and 𝐵 = ( ). [3]
2 1 2 5

1 2 1 3
𝐴𝐵 = ( )( )
2 1 2 5

(1 × 1) + (2 × 2) (1 × 3) + (2 × 5)
𝐴𝐵 = ( )
(2 × 1) + (1 × 2) (2 × 3) + (1 × 5)

1 + 4 3 + 10
𝐴𝐵 = ( )
2+2 6+5

5 13
𝐴𝐵 = ( )
4 11

Now,

5 13
3𝐴𝐵 = 3 ( )
4 11

15 39
3𝐴𝐵 = ( )
12 33

(b) The diagram below, not drawn to scale, shows a triangle, 𝐴𝐵𝐶 whose coordinates are

stated.

Triangle 𝐴𝐵𝐶 undergoes two successive transformations, 𝑉 followed by 𝑊, where

2 0 −1 0
𝑉=( ) and 𝑊 = ( ).
0 2 0 1

287
(i) State the effect of 𝑉 on triangle 𝐴𝐵𝐶. [2]

The image of triangle 𝐴𝐵𝐶 under 𝑉 is an enlargement, centre 𝑂 and scalar

factor 2.

(ii) Determine the 2 × 2 matrix that represents the combined transformation of 𝑉

followed by 𝑊. [3]

The combined transformation 𝑉 followed by 𝑊 is expressed as 𝑊𝑉.

−1 0 2 0
𝑊𝑉 = ( )( )
0 1 0 2

(−1 × 2) + (0 × 0) (−1 × 0) + (0 × 2)
𝑊𝑉 = ( )
(0 × 2) + (1 × 0) (0 × 0) + (1 × 2)

−2 + 0 0 + 0
𝑊𝑉 = ( )
0+0 0+2

−2 0
𝑊𝑉 = ( )
0 2

(iii) Use your matrix in (b)(ii), determine the coordinates of the image of triangle

𝐴𝐵𝐶 under this combined transformation. [3]

The coordinates of triangle 𝐴𝐵𝐶 are 𝐴(1, 2), 𝐵(1, 1) and 𝐶(2, 1).

Image of 𝐴(1, 2):

−2 0 1 (−2 × 1) + (0 × 2)
( )( ) = ( )
0 2 2 (0 × 1) + (2 × 2)

−2 0 1 −2 + 0
( )( ) = ( )
0 2 2 0+4

−2 0 1 −2
( )( ) = ( )
0 2 2 4

The coordinates of 𝐴′ (−2, 4).

288
Image of 𝐵(1, 1):

−2 0 1 (−2 × 1) + (0 × 1)
( )( ) = ( )
0 2 1 (0 × 1) + (2 × 1)

−2 0 1 −2 + 0
( )( ) = ( )
0 2 1 0+2

−2 0 1 −2
( )( ) = ( )
0 2 1 2

The coordinates of 𝐵 ′ (−2, 2).

Image of 𝐶(2, 1):

−2 0 2 (−2 × 2) + (0 × 1)
( )( ) = ( )
0 2 1 (0 × 2) + (2 × 1)

−2 0 2 −4 + 0
( )( ) = ( )
0 2 1 0+2

−2 0 2 −4
( )( ) = ( )
0 2 1 2

The coordinates of 𝐶 ′ (−4, 2).

Therefore,

𝐴(1, 2) is mapped onto 𝐴′ (−2, 4).

𝐵(1,1) is mapped onto 𝐴′ (−2, 2).

𝐶(2,1) is mapped onto 𝐴′ (−4, 2).

289
(c) (i) Write the following simultaneous equations in the form 𝐴𝑋 = 𝐵 where 𝐴, 𝑋 and 𝐵

are matrices:

11𝑥 + 6𝑦 = 6

9𝑥 + 5𝑦 = 7 [2]

Change to matrix form:

11 6 𝑥 6
( ) (𝑦 ) = ( )
9 5 7

in the form 𝐴𝑋 = 𝐵

∴ 𝑋 = 𝐴−1 𝐵

11 6
where 𝐴 = ( ).
9 5

(ii) Hence, write the solution for 𝑥 and 𝑦 as a product of two matrices. [2]

Find 𝐴−1:

𝑑 −𝑏
det (𝐴) = 𝑎𝑑 − 𝑏𝑐 𝑎𝑑𝑗(𝐴) = ( )
−𝑐 𝑎

5 −6
= (11)(5) − (6)(9) =( )
−9 11

= 55 − 54

=1

1
𝐴−1 = 𝑑𝑒𝑡 (𝑎𝑑𝑗)

1 5 −6
= 1( )
−9 11

290
So, by rearranging, we have:

𝑥 1 5 −6 6
(𝑦) = 1 ( )( )
−9 11 7
𝑥 30 − 42
(𝑦 ) = ( )
−54 + 77
𝑥 −12
(𝑦 ) = ( )
23

∴ 𝑥 = −122 and 𝑦 = 23

Total: 15 marks

291
Past Paper Question – June 2023 – Paper 2 – Question 10(a) and (b)

10. (a) The matrices 𝑨 and 𝑩 represent the transformations given below.

0 −1
𝐴=( ) represents an anticlockwise rotation of 90° about the origin, 𝑂.
1 0

0 −1
𝐵=( ) represents a reflection in the straight line with equation
−1 0

𝑦 = −𝑥.

(i) Determine the elements of the matrix 𝑪 which represents an

anticlockwise rotation of 90° about the origin, 𝑂, followed by a

reflection in the straight line 𝑦 = −𝑥. [2]

(ii) Describe, geometrically, the single transformation represented

by 𝑪. [2]

(b) A transformation, 𝑇, is defined by the following 2 × 2 matrix.

1 2
𝑇=( ) , where 𝑘 is a constant.
𝑘 −1

𝑇 maps the point (2, 3) onto the point (8, 15).

Determine the value of 𝑘. [2]

292
Past Paper Question – June 2023 – Paper 2 – Question 10(a) and (b) – Solution

10. (a) The matrices 𝑨 and 𝑩 represent the transformations given below.

0 −1
𝐴=( ) represents an anticlockwise rotation of 90° about the origin, 𝑂.
1 0

0 −1
𝐵=( ) represents a reflection in the straight line with equation
−1 0

𝑦 = −𝑥.

(i) Determine the elements of the matrix 𝑪 which represents an

anticlockwise rotation of 90° about the origin, 𝑂, followed by a

reflection in the straight line 𝑦 = −𝑥. [2]

𝐶 = 𝐵𝐴

0 −1 0 −1
𝐶=( )( )
−1 0 1 0

(0 × 0) + (−1 × 1) (0 × −1) + (−1 × 0)


𝐶=( )
(−1 × 0) + (0 × 1) (−1 × −1) + (0 × 0)

0 + (−1) 0 + 0
𝐶=( )
0+0 1+0

−1 0
𝐶=( )
0 1

(ii) Describe, geometrically, the single transformation represented

by 𝑪. [2]

The matrix 𝐶 represents a reflection in the 𝑦-axis.

293
(b) A transformation, 𝑇, is defined by the following 2 × 2 matrix.

1 2
𝑇=( ) , where 𝑘 is a constant.
𝑘 −1

𝑇 maps the point (2, 3) onto the point (8, 15).

Determine the value of 𝑘. [2]

𝑇𝑃 = 𝑃′

1 2 2 8
( )( ) = ( )
𝑘 −1 3 15

Using the last row,

2𝑘 + (−3) = 15

2𝑘 − 3 = 15

2𝑘 = 15 + 3

2𝑘 = 18

18
𝑘=
2

𝑘=9

294

You might also like